General Physics 1 - LAS - QTR1 - STEM - STUDENTS

Download as pdf or txt
Download as pdf or txt
You are on page 1of 57

WEEKLY LEARNING ACTIVITY SHEETS

General Physics I 12 Quarter 1 Week 1

UNITS AND MEASUREMENT

Name: ___________________________________________ Section: ____________________

Most Essential Learning Competencies


 Solve measurement problems involving conversion of units, expression of measurements
in scientific notation. (STEM_GP12EU-Ia-1)
 Differentiate accuracy from precision. (STEM_GP12EU-Ia-2)
 Differentiate random errors from systematic errors. (STEM_GP12EU-Ia-3)
 Estimate errors from multiple measurements of a physical quantity using variance.
(STEM_GP12EU-Ia-5)
 Differentiate vector and scalar quantities. (STEM_GP12EU-Ia-8)
 Perform addition of vectors. (STEM_GP12EU-Ia-9)
 Rewrite a vector in component form. (STEM_GP12EU-Ia-10)

Specific Objectives
 Convert units of measurement
 Express numbers in scientific notation.
 Apply the rules of counting significant figures in rounding-off numbers based on the
mathematical operation used.
 Determine the number of significant figures.
 Differentiate accuracy from precision.
 Distinguish between scalar and vector quantities, and random and systematic errors.
 Add vectors using graphical and component method.

Time Allotment: 4 hours

Key Concepts

Units of measurements
 Physicists, like other scientists, make observations and ask basic questions. For
example, how big is an object? How much mass does it have? How far did it travel? To
answer these questions, they make measurements with various instruments (e.g., meter
stick, balance, stopwatch, etc.).
 The measurements of physical quantities are expressed in terms of units, which are
standardized values. For example, the length of a race, which is a physical quantity, can
be expressed in meters (for sprinters) or kilometers (for long distance runners). Without
standardized units, it would be extremely difficult for scientists to express and compare
measured values in a meaningful way.

Physical Quantities
 Any number that is used to describe a physical phenomenon quantitatively is called
physical quantity.
 We define a physical quantity either by specifying how it is measured or by stating how
it is calculated from other measurements.
 For instance, you can measure the distance an object travels by using a meterstick and
you can measure the time interval by using a stopwatch.

Author: RIC ME D. DIAZ


School/Station: LIANGA NATIONAL COMPREHENSIVE HIGH SCHOOL
Division: SURIGAO DEL SUR
email address: ricme.diaz@deped.gov.ph
 There are two major systems of units that are used in the world: (1) Metric System is
commonly used by scientists and engineers around the world, but since 1960 it has
been officially known as the International System of Units or SI units.(2) English
System of Units, also known as the British units, were historically used in nations once
ruled by the British Empire and are still widely used in the United States of America.
English system of units may also be referred to as the foot-pound-second or fps system,
as opposed to the centimeter-gram-second or cgs system.

SI units: Fundamental and Derived Units


 Some physical quantities are more fundamental than others. In physics, there are seven
fundamental physical quantities that are measured in base or physical fundamental
units: length, mass, time, electric current temperature, amount of substance, and
luminous intensity.
 The units in which they are measured are the meter, kilogram, second, ampere, kelvin,
mole, and candela. All other units are made by mathematically combining the
fundamental units. These are called derived units.
Table 1. SI Base Units
Quantity Name Symbol
Length Meter M
Mass Kilogram Kg
Time Second S
Electric current Ampere A
Temperature Kelvin K
Amount of substance Mole Mol
Luminous intensity Candela Cd

Metric Prefixes
 Physical objects or phenomena may vary widely. For example, the size of objects varies
from something very small (like an atom) to something very large (like a star). Yet the
standard metric unit of length is the meter. So, the metric system includes many prefixes
that can be attached to a unit. Each prefix is based on factors of 10 (10, 100, 1,000,
etc., as well as 0.1, 0.01, 0.001, etc.).
Table 2. Metric Prefixes and symbols used to denote the different various factors of 10 in the metric system
Example Example Example
Prefix Symbol Value Example Description
Name Symbol Value
Distance light travels in a
Exa E 1018 Exameter Em 1018 m
century
Peta P 1015 Petasecond Ps 1015 s 30 million years
Tera T 1012 Terawatt TW 1012 W Powerful laser output
Giga G 109 Gigahertz GHz 109 Hz A microwave frequency
Mega M 106 Megacurie MCi 106 Ci High radioactivity
Kilo K 103 Kilometer Km 103 m About 6/10 mile
hector H 102 Hectoliter hL 102 L 26 gallons
Deka Da 101 Dekagram Dag 101 g Teaspoon of butter
____ ____ 100 (=1)
Deci D 10–1 Deciliter dL 10–1 L Less than half a soda
Centi C 10–2 Centimeter Cm 10–2 m Fingertip thickness
Mili M 10–3 Millimeter Mm 10–3 m Flea at its shoulder
Micro µ 10–6 Micrometer µm 10–6 m Detail in microscope
Nano N 10–9 Nanogram Ng 10–9 g Small speck of dust
Pico P 10–12 Picofarad pF 10–12 F Small capacitor in radio
Femto F 10–15 Femtometer Fm 10–15 m Size of a proton
Atto A 10–18 Attosecond As 10–18 s Time light takes to cross an atom

Author: RIC ME D. DIAZ


School/Station: LIANGA NATIONAL COMPREHENSIVE HIGH SCHOOL
Division: SURIGAO DEL SUR
email address: ricme.diaz@deped.gov.ph
Unit Conversion and Dimensional Analysis
 It is often necessary to convert from one unit to another. Some applications require the
conversion from SI unit to English unit and vice versa. In conversion, we need to
establish the conversion factor.
 It is a ratio that expresses the relationship between two units, i.e. how many or how
much of one unit is equal to another unit. Since the two different values mean the same
thing, conversion factor is just equal to one (1). Conversion of English units to SI units
have been defined exactly as follows:
Length: 1 foot(ft) = 12 inches(in) ; 1 in = 2.54 cm ; 1 mile (mi) = 1.609 km
Mass: 1 pound-mass (lbm) = 0.4536 kg
Time: Second is the same for both the English and the SI system of units.
Force: 1 pound-force (lbf) = 4.448 newtons (N)
Note: Refer to Appendix B for more list of conversion factors.
 Conversion uses dimensional analysis method of solving problems. Dimensional
analysis involves cancelation of the given unit(s) from the given quantity and conversion
factor(s) to get the desired units(s). Mathematically, it may be written as

𝑔𝑖𝑣𝑒𝑛 𝑞𝑢𝑎𝑛𝑡𝑖𝑡𝑦 × 𝑐𝑜𝑛𝑣𝑒𝑟𝑠𝑖𝑜𝑛 𝑓𝑎𝑐𝑡𝑜𝑟 = 𝑑𝑒𝑠𝑖𝑟𝑒𝑑 𝑞𝑢𝑎𝑛𝑡𝑖𝑡𝑦

𝑑𝑒𝑠𝑖𝑟𝑒𝑑 𝑢𝑛𝑖𝑡
𝑔𝑖𝑣𝑒𝑛 𝑢𝑛𝑖𝑡 × = 𝑑𝑒𝑠𝑖𝑟𝑒𝑑 𝑢𝑛𝑖𝑡
𝑔𝑖𝑣𝑒𝑛 𝑢𝑛𝑖𝑡

 Example, we want to convert one’s height of 1.5 m to feet. The first thing to do is to know
the given unit and the desired unit. In this case, the given unit is in meters and it is to
be converted to feet. Next, we need to determine the conversion factor relating meters to
feet. From the given conversion factors above for length, there is no direct conversion
from meters to feet. Thus, meters must be converted first to centimeters, then convert
centimeters to inches, and finally convert inches to feet. Therefore, 1.5 m can be
converted to feet as follows

100 𝑐𝑚 1 𝑖𝑛 1 𝑓𝑡 1.5 × 100


1.5 𝑚 × × × = 𝑓𝑡 = 4.9 𝑓𝑡.
1𝑚 2.54 𝑐𝑚 12 𝑖𝑛 2.54 × 12

Example 2. Converting speed units


The world land speed record is 763.0 mi/h, set on October 15, 1997, by Andy Green in
the jet-engine car Thrust SSC. Express this speed in meters per second.

Solution: We need to convert the units of a speed from mi/h to m/s. We must therefore
find conversion factors that relate (i) miles to meters and (ii) hours to seconds. In
Appendix B, we find the conversion factors 1 mi = 1.609 km, 1 km = 1000 m, and 1 h =
3600 s. We set up the conversion as follows, which ensures that all the desired
cancellations by division take place:

𝑚𝑖 𝑚𝑖 1.609 𝑘𝑚 1000 𝑚 1ℎ 𝑚
763.0 = 763.0 × × × = 341.0
ℎ ℎ 1 𝑚𝑖 1 𝑘𝑚 3600 𝑠 𝑠

Rounding off Numbers and Significant Figures


 One reason why we round off numbers is to simplify a report of a measurement or a
calculation.
 For a single value, just know which last digit to keep. This last digit remains the same if
the next digit to the right is less than 5, but if it is greater than or equal to 5, the last
digit increases by 1.

Author: RIC ME D. DIAZ


School/Station: LIANGA NATIONAL COMPREHENSIVE HIGH SCHOOL
Division: SURIGAO DEL SUR
email address: ricme.diaz@deped.gov.ph
 For instance, if we round off 343 and 245 to the nearest tens, the answer is 340 and
250, respectively. Similarly, if we round off 8.345 and 67.47 to the nearest tenths, the
answer is 8.3 and 67.5, respectively.
 In measurement, however, what dictates which last digit to keep in reporting final
answers follows another guideline, depending on the mathematical operation involved.
 For addition or subtraction, count the number of decimal places each measurement in
the calculation has. The answer cannot have more digits to the right of the decimal point
than any of the original numbers. The result has the same number of decimal places as
the least precise measurement used in the calculation.

Example. As an illustration, what would be the result if 1.1 is added to 89.332?

89.332 ← 3 decimal places


+ 1.1 ← 1 decimal place (least precise)
90.432 ← round off to 90.4

 For multiplication or division, count the number of significant figures each


measurement in the calculation has. Significant figures are the reliably known digits.
The number of significant figures in the result is set by the original number that has the
least number of significant figures. How do we count the number of significant figures?
The following are the guidelines in determining whether a digit is significant or not.

1. All nonzero digits are significant. (nonzero integers)


o 1.234 kg → 4 sig. fig., 24 mL → 2 sig. fig., 6.4x104 molecules → 2 sig. fig.
2. Zeros between nonzero digits are significant. (captive zeros)
o 606 m → 3 sig. fig., 3001 g → 4 sig. fig., 10101 s → 5 sig. fig.
3. Zeros to the left of the first nonzero digit are not significant. (leading zeros)
o 0.08 L → 1 sig. fig., 0.0055 mol → 2 sig. fig., 0.0234 km → 3 sig. fig.
4. Zeros to the right of a nonzero digit and are on the right side of the decimal point are
significant.
o 2.00 mg → 3 sig. fig., 0.00420 g → 3 sig. fig., 6.022x10 23 atoms → 4 sig. fig.
5. Trailing zeros in a whole number are not significant. (ambiguous zeros)
o 560 kg → 2 sig. fig., 1500 mL → 2 sig. fig., 1250 m → 3 sig. fig.
6. Exact numbers, or numbers from definition or number of objects, are considered to have
an infinite number of significant figures.
o 1 kg, 2.54 cm (exactly) or 1 inch, 299792458 m/s or the speed of light, 2 pencils
Note that in conversion problems, the conversion factor consists exact numbers.
Since exact numbers contain infinite number of significant figures, the basis in
identifying the measurement having the least number of significant figures is the
given or original value.

Example. Find the product of 3.666 and 4.51.

3.666 → 4 significant figures


x 4.51 → 3 significant figures (least number of sig. fig.)
16.536366 → round off to 16.5

Author: RIC ME D. DIAZ


School/Station: LIANGA NATIONAL COMPREHENSIVE HIGH SCHOOL
Division: SURIGAO DEL SUR
email address: ricme.diaz@deped.gov.ph
Scientific Notation
 When we calculate with very large or very small numbers, we can show significant
figures much more easily by using scientific notation, sometimes called powers-of-10
notation. Scientific notation is a convenient way of expressing too large or too small
numbers. It is called powers-of-10 because it follows the form

𝑁 × 10𝑛
where,
 𝑁 is a number between 1 and 10, including 1 but not 10, i.e. 1 ≤ |𝑁 | < 10;
Note that for the exponent 𝑛:
 If the decimal point is moved 𝑛 places to the left, 𝑛 is positive.
 If the decimal point is moved 𝑛 places to the right, 𝑛 is negative.

Example. The distance from the Earth to the moon is about 384,000,000 m. To write
this number in scientific notation, we move the decimal point eight places to the left,
that is,
3 8 4, 0 0 0, 0 0 0. m → 3 . 8 4 0 0 0 0 0 0 x 108 m = 3.84 x108 m
↑ ↑ ↑ ↑ ↑ ↑ ↑ ↑ ↑
start 8 7 6 5 4 3 2 1 ← to the left

In this form, it is clear that we have 3 significant figures. It is important to note that in
counting the number of significant figures in the form of scientific notation only the “𝑁”
part should be considered, not the “× 10𝑛 ” part, following the same guidelines.

 When carrying out arithmetic operations involving scientific notation, same rules in
rounding-off final answers should be applied. Addition or subtraction follows the value
with the least number of decimal places while multiplication or division follows the value
with the least number of significant figures. Consider the examples and steps below.

For addition or subtraction: Consider adding 4.31x104 and 3.9x103.


Step 1: Write each quantity with the same exponent n. → 4.31x104 + 0.39x104
Step 2: Combine N1 and N2. = (4.31+0.39) x104
Step 3: The exponent, n, remains the same. = 4.70x104

For multiplication: Consider 4.0x10-5 x 7.0x103.


Step 1: Multiply N1 and N2. → (4.0 x 7.0) x (10-5 x 103)
Step 2: Add the exponents n1 and n2. = 28x10-5+3 = 28x10-2 = 2.8x10-1

For division: Consider 8.5x104 ÷ 5.0x109.


Step 1: Divide N1 and N2. → (8.5÷5.0) x (104 x 109)
Step 2: Subtract exponents n1 and n2. = 1.7x104-9 = 1.7x10-5

Author: RIC ME D. DIAZ


School/Station: LIANGA NATIONAL COMPREHENSIVE HIGH SCHOOL
Division: SURIGAO DEL SUR
email address: ricme.diaz@deped.gov.ph
Accuracy and Precision
 Accuracy tells us how close the measurement is to the true value of the quantity that was
measured. Precision, on the other hand, refers to how close a set of measurements of
the same quantity agree with one another.
 Consider the figure below, the distribution of holes formed by darts shows as an analogy
that a set of measurement is (a) accurate and precise. (b) precise but not accurate. (c)
not accurate and not precise.

(a) (b) (c)

Figure 1. Accuracy and Precision


Source: Young, Hugh D., Roger A. Freedman, A. Lewis Ford, and Hugh D. Young. Sears and Zemansky's University
Physics. (13th ed), p. 1124

 An important factor in the precision of measurements involves the precision of the


measuring tool. In general, a precise measuring tool is one that can measure values in
very small increments.
 For example, a standard ruler can measure length to the nearest millimeter whereas a
Vernier caliper can measure length to the nearest 0.01 mm. The caliper is a more precise
measuring tool than a ruler because it can measure extremely small differences in
length. The more precise the measuring tool, the more precise the measurements.
 To ensure high accuracy and precision, one consideration is using appropriate
measuring instruments designed to fit the purpose. For example; if you want to measure
a piece of wood, tape measure is suitable.

Random and Systematic Errors


 No matter how we carefully perform the measurement, error is always inevitable. Note
that error is not a “mistake” – it’s part of the measuring process. In science,
measurement error is called experimental error or observational error.
 Two broad classifications of errors are random error and systematic error.
 Random error varies unpredictably from one measurement to another, while systematic
error has the same value or proportion for every measurement. Random errors are
unavoidable, but cluster around the true value.
 Random error primarily affects precision. Typically, random error affects the last
significant digit of a measurement. The main reasons for random error are limitations of
instruments, environmental factors, and slight variations in procedure. For example:
o When weighing yourself on a bathroom scale, you position yourself slightly
differently each time.
o When taking a volume reading in a flask, you may read the value from a different
angle each time.
o Measuring the mass of a sample on an analytical balance may produce different
values as air currents affect the balance.
o Measuring your height is affected by minor posture changes.
 Systematic error can often be avoided by calibrating equipment, but if left uncorrected,
can lead to measurements far from the true value.
 Systematic error is predictable and either constant or else proportional to the
measurement. Systematic errors primarily influence a measurement's accuracy. Typical
causes of systematic error include observational error, imperfect instrument calibration,
and environmental interference. For example:

Author: RIC ME D. DIAZ


School/Station: LIANGA NATIONAL COMPREHENSIVE HIGH SCHOOL
Division: SURIGAO DEL SUR
email address: ricme.diaz@deped.gov.ph
o Forgetting to tare or zero a balance produces mass measurements that are always
"off" by the same amount. An error caused by not setting an instrument to zero
prior to its use is called an offset error.
o Not reading the meniscus at eye level for a volume measurement will always result
in an inaccurate reading. The value will be consistently low or high, depending
on whether the reading is taken from above or below the mark.
o Measuring length with a metal ruler will give a different result at a cold
temperature than at a hot temperature, due to thermal expansion of the material.
o An improperly calibrated thermometer may give accurate readings within a
certain temperature range but become inaccurate at higher or lower
temperatures.
o Measured distance is different using a new cloth measuring tape versus an older,
stretched one. Proportional errors of this type are called scale factor errors.

Scalar and Vector Quantities


 It is useful to separate physical quantities into two types: the first are called scalars; the
second are known as vectors. A scalar is a quantity that can be described by a single
number which can be positive, negative or zero.
 An example of a scalar quantity is the mass of an object, so we might state that ‘the
mass of the stone is 3 kg’. It is important to give the units in which the quantity is
measured.
 On the other hand, vectors are
quantities which require the
specification of a magnitude and a
direction. An example of a vector
quantity is the force applied to an Figure 2. Force is a vector quantity.
object to make it move. When the object shown in Figure 2 is moved by applying a force
to it, we achieve different effects depending on the direction of the force.

Vector Addition
 Calculations that combine scalar quantities use the operations of ordinary arithmetic.
For example, 6 kg + 3 kg = 9 kg or 4 x 2 s = 8 s. However, combining vectors requires a
different set of operations.
 To understand more about vectors and how they combine, we start with the simplest
vector quantity, displacement. Displacement is simply a change in the position of an
object.
 Displacement is a vector quantity because we must state not only how far the object
moves but also in what direction. Walking 3 km north from your front door doesn’t get
you to the same place as walking 3 km southeast; these two displacements have the
same magnitude but different directions.
 We usually represent a vector quantity such as displacement by a single letter, 𝐴⃗ such
as in Figure 3a. Displacement is always a straight-line segment directed from the
starting point to the ending point, even though the object’s actual path may be curved
(Figure 3b).
 Note that displacement is not related directly to the total distance traveled. If the object
were to continue on past P2 and then return to P1, the displacement for the entire trip
would be zero (Figure 3c).

Figure 3. Displacement is a vector


quantity.
Source: Young, Hugh D., Roger A.
Freedman, A. Lewis Ford, and
Hugh D. Young. Sears and
Zemansky's University Physics.
(13th ed), p. 11

Author: RIC ME D. DIAZ


School/Station: LIANGA NATIONAL COMPREHENSIVE HIGH SCHOOL
Division: SURIGAO DEL SUR
email address: ricme.diaz@deped.gov.ph
 Suppose a particle undergoes a displacement ⃗𝑨 ⃗⃗ followed by a second displacement ⃗𝑩 ⃗⃗.
The final result is the same as if the particle had started at the same initial point and
undergone a single displacement ⃗𝑪⃗ (Figure 4a).
 We call displacement 𝑪 ⃗⃗ the vector sum, or resultant, of displacements 𝑨 ⃗⃗⃗ and 𝑩⃗⃗⃗. We
express this relationship symbolically as 𝑪 ⃗⃗ = 𝑨
⃗⃗⃗ + 𝑩⃗⃗⃗.
 The boldface plus sign emphasizes that adding two vector quantities requires a
geometrical process and is not the same operation as adding two scalar quantities such
as 2 +3 = 5. In vector addition we usually place the tail of the second vector at the head,
or tip, of the first vector (Figure 4a).
 Figure 4c shows another way to represent the vector sum: If vectors ⃗𝑨 ⃗⃗ and ⃗𝑩⃗⃗ are both
drawn with their tails at the same point, vector ⃗𝑪⃗ is the diagonal of a parallelogram
constructed with 𝑨 ⃗⃗⃗ and 𝑩
⃗⃗⃗ as two adjacent sides.

Figure 4. Three ways to add vectors.


Source: Young, Hugh D., Roger A. Freedman, A. Lewis Ford, and Hugh D. Young. Sears and Zemansky's University
Physics. (13th ed), p. 12

 ⃗⃗ = 𝑨
Several constructions for finding the vector sum 𝑪 ⃗⃗⃗ + 𝑩
⃗⃗⃗.

Figure 5. Other ways to add vectors.


Source: Young, Hugh D., Roger A. Freedman, A. Lewis Ford, and Hugh D. Young. Sears and Zemansky's University
Physics. (13th ed), p. 12

Example. A cross-country skier skis 1.00 km north


and then 2.00 km east on a horizontal snowfield. How
far and in what direction is she from the starting point?
Solution.
1. In this case, vector addition amounts to solving a
right triangle, which we can do using the
Pythagorean theorem and simple trigonometry.
2. The distance from the starting point to the ending
point is equal to the length of the hypotenuse:
𝑐 = √𝑎2 + 𝑏2
𝑐 = √(1.00 𝑘𝑚)2 + (2.00 𝑘𝑚)2 Figure 6. The vector diagram, drawn to
scale for a ski trip.
𝑐 = 2.24 𝑘𝑚
3. A little trigonometry allows us to find angle θ:
𝑜𝑝𝑝𝑜𝑠𝑖𝑡𝑒 𝑠𝑖𝑑𝑒 2.00 𝑘𝑚
𝑡𝑎𝑛 𝜃 = = = 63.4°
𝑎𝑑𝑗𝑎𝑐𝑒𝑛𝑡 𝑠𝑖𝑑𝑒 1.00 𝑘𝑚
We can describe the direction as 63.4° east of north or 90° - 63.4° = 26.6° north of east.

Author: RIC ME D. DIAZ


School/Station: LIANGA NATIONAL COMPREHENSIVE HIGH SCHOOL
Division: SURIGAO DEL SUR
email address: ricme.diaz@deped.gov.ph
Components of Vectors
 A coordinate system allows
you to expand your
description of a vector. In
coordinate system shown in
Figure 7, the vector ⃗𝑨 ⃗⃗ is
broken up into two
component vectors. One, ⃗𝑨 ⃗⃗𝑥 ,
is parallel to the x-axis, and
Figure 7. Representing a vector in terms of component vectors.
the other, ⃗𝑨
⃗⃗𝑦 , is parallel to the Source: Young, Hugh D., Roger A. Freedman, A. Lewis Ford,
y-axis. In symbols, ⃗𝑨 ⃗⃗ = ⃗𝑨
⃗⃗𝑥 + ⃗𝑨⃗⃗𝑦 . and Hugh D. Young. Sears and Zemansky's University
Physics. (13th ed), p. 14

 The process of breaking a vector in to its components is


called vector resolution. The magnitude and sign of the
component vectors are called the components.
 The components are calculated according to these
equations shown below, where the angle is measured
counterclockwise from the positive axis.

Figure 8. The sign of a component


Figure 7. Calculating vector components. depends upon which of the quadrants
the component is in.
Example. A bus travels 23.0 km on a straight road that is 30° north of east.
What are the east and north components of its displacement?

Solution. Use the trigonometric ratios to find the components.


⃗𝑨 ⃗⃗𝑥 = ⃗𝑨
⃗⃗ cos 𝜃 ⃗𝑨 ⃗⃗𝑦 = ⃗𝑨
⃗⃗ sin 𝜃
⃗𝑨⃗⃗𝑥 = (23.0 𝑘𝑚) cos 30° ⃗𝑨⃗⃗𝑦 = (23.0 𝑘𝑚) sin 30°
⃗𝑨
⃗⃗𝑥 = 19.9 𝑘𝑚 ⃗𝑨
⃗⃗𝑦 = 11.5 𝑘𝑚

Author: RIC ME D. DIAZ


School/Station: LIANGA NATIONAL COMPREHENSIVE HIGH SCHOOL
Division: SURIGAO DEL SUR
email address: ricme.diaz@deped.gov.ph
Activity 1: Conversion of Units, Scientific Notation, and Significant Figures

Objectives
 Convert units of measurement
 Express numbers in scientific notation.
 Apply the rules of counting significant figures in rounding-off numbers based on the
mathematical operation used.
 Determine the number of significant figures.

What you need


 pencil/ballpen
 sheets of paper
 scientific calculator

What to do
 1A. Determine the number of significant figures in the following measurements:
(a) 5.03 g (f) 2.720 x 1022 atoms
(b) 0.714 m (g) 3 000 mL
(c) 0.052 kg (h) 35 mg
(d) 2008 g (i) 0.0580 m3
(e) 7.2 x 10 molecules
4 (j) 830 kg

 1B. Express the following number in standard notation to scientific notation. Answers
should include 3 significant figures applying rules in rounding-off numbers.

(a) 75 000 (c) 1 647 000 000


(b) 0.004894 (d) 0.000001995

 1C. Carry out the following arithmetic operations applying the rules on rounding-off final
answers.

(a) 12 343.2 g + 0.1893 g - 78.238 g


(b) 55.67 L – 2.386 L + 4.3237 L
(c) 7.52 kg x 6.9232 m/s2
(d) 0.0239 kg ÷ 46.5 mL
(e) 5.21 x 103 + 2.92 x 102
(f) 3.62 x 10-5 – 5.14 x 10-6
(g) 7.1 x 104 x 2.2654 x 102
(h) 7.50 x 10-7 ÷ 2.20 x 10-13

 1D. Convert the following quantities as indicated. If possible, express answers in scientific
notation.

(a) 365 days to minutes (e) 1.08x108 dm3 to m3


(b) 107.5 MHz to kHz (f) 0.0833 lbm to mg
(c) 650 cm2 to km2 (g) 0.808 g/cm3 to kg/m3
(d) 1000 L to m3 (Hint: 1 mL = 1 cm3) (h) 75 mph to m/s

Author: RIC ME D. DIAZ


School/Station: LIANGA NATIONAL COMPREHENSIVE HIGH SCHOOL
Division: SURIGAO DEL SUR
email address: ricme.diaz@deped.gov.ph
Activity 2: Accuracy and Precision

Objective
 Differentiate accuracy from precision.

What you need


 pencil/ballpen
 sheets of paper
 scientific calculator

What to do
 Read and answer as directed. Write your answers in a separate sheet of paper.

 Gold coin has an ‘accepted’ diameter of 28.054 mm. Two students are asked to measure
the diameter a gold coin four times. Student A uses a simple plastic ruler. Student B uses
a precision measuring tool called a micrometer.

Student A (Plastic ruler) Student B (Micrometer)


27.9 mm 28.246 mm
28.0 mm 28.244 mm
27.8 mm 28.246 mm
28.1 mm 28.248 mm

 Calculate the average diameter for each set of measurement.

Student A (Plastic ruler) Student B (Micrometer)

 Compare the average value for each set with the accepted value.
1. Which student’s data is more accurate? Justify your answer.

2. Which student’s data is more precise? Justify your answer.

Activity 3: Which is which?


Scalar and Vector Quantities, Random and Systematic Errors

Objective
 Distinguish between scalar and vector quantities, and random and systematic errors.
What you need
 pencil/ballpen
 sheets of paper
 scientific calculator

What to do

 3A. In a separate sheet of paper, write SCALAR if the given is a scalar quantity and write
VECTOR if the given is a vector quantity.
1. Area of a rectangle
2. Distance

Author: RIC ME D. DIAZ


School/Station: LIANGA NATIONAL COMPREHENSIVE HIGH SCHOOL
Division: SURIGAO DEL SUR
email address: ricme.diaz@deped.gov.ph
3. Work
4. Increase/Decrease in temperature
5. Magnetic Polarization
6. Momentum
7. Temperature
8. Force
9. Energy
10. Density

 3B. In a separate sheet of paper, write RANDOM if the given is a random error and write
SYSTEMATIC if the given is a systematic error.
1. When taking a volume reading in a flask, you may read the value from a different
angle each time.
2. Measuring the mass of a sample on an analytical balance may produce different
values as air currents affect the balance.
3. Measured distance is different using a new cloth measuring tape versus an older,
stretched one.
4. When weighing yourself on a bathroom scale, you position yourself slightly differently
each time.
5. Forgetting to tare or zero a balance produces mass measurements that are always
"off" by the same amount.
6. Not reading the meniscus at eye level for a volume measurement.
7. When measuring participants’ wrist circumferences, you misread the “2” on the
measuring tape as a zero-point.
8. Using an improperly calibrated thermometer.
9. You measure wrist circumference using a tape measure. But your tape measure is
only accurate to the nearest half-centimeter, so you round each measurement up or
down when you record data.
10. If a piece of equipment has an offset.

Activity 4: Addition of Vectors

Objective
 Add vectors using graphical and component method.
What you need
 pencil/ballpen
 ruler and protractor
 sheets of paper
 scientific calculator

What to do

 3A. In a separate sheet, add the following vectors using graphical method and determine
the magnitude of the resultant vector and its direction.
1. A. 5.0 km, West
B. 2.5 km, North (Scale: 1 cm = 1 km)
C. 3.5 km, 345°

2. A. 8 km, 150°
B. 6 km, 220° (Scale: 1 cm = 1 km)
C. 5 km, 290°

Author: RIC ME D. DIAZ


School/Station: LIANGA NATIONAL COMPREHENSIVE HIGH SCHOOL
Division: SURIGAO DEL SUR
email address: ricme.diaz@deped.gov.ph
 3B. In a separate sheet, add the following vectors using analytical method and determine
the magnitude of the resultant vector and its direction.
1. A. 4.5 km, 20°
B. 4.2 km, 270°
C. 6.0 km, 210°

2. A. 25 N, 60°
B. 10 N, 180°
C. 16 N, 315°

Reflection

Directions: In a separate sheet of paper, write a five-sentence explanation of the quote


given below.

“Take care of your units and they will take care of you” ~ David M. Himmelblau

Rubrics
5 4 3 2 0
Practical Practical Practical Practical No discussion at
application is application is application is applications are all.
scientifically scientifically explained explained
explained explained consistent to the consistent to the
consistent to the consistent to the concepts but concepts but
concepts, and concepts, but with one or two with more than
has no with minimal misconceptions. two
misconceptions. misconceptions. misconceptions.

References for learners:

Krauskopf K., & Beiser A. (2010). The Physical Universe (14th ed.). The McGraw-Hill Companies,
Inc., 1221 Avenue of the Americas, New York, NY 10020.

Shipman, J., Wilson, J., & Higgins, C. (2013). An Introduction to Physical Science (13 th ed.).
Brooks/Cole, Cengage Learning

Young, Hugh D., Roger A. Freedman, A. Lewis Ford, and Hugh D. Young. Sears and Zemansky's
University Physics. 13th ed. Boston, MA: Pearson Learning Solutions, 2012.

Author: RIC ME D. DIAZ


School/Station: LIANGA NATIONAL COMPREHENSIVE HIGH SCHOOL
Division: SURIGAO DEL SUR
email address: ricme.diaz@deped.gov.ph
email address: ricme.diaz@deped.gov.ph
Division: SURIGAO DEL SUR
School/Station: LIANGA NATIONAL COMPREHENSIVE HIGH SCHOOL
Author: RIC ME D. DIAZ
Activity 4: Addition of Vectors
4A
1. R ≈ 2.3 km, 135° CCW from E
2. R ≈ 10.8 km, 205° CCW from E Activity 2: Accuracy and
Precision
4B
1. R = 5.74 km, 260.3° CCW from E (a) Student A – 28.0
2. R = 17. 3 N, 37° N of E Student B – 28.246
(b) more accurate – Student A
Activity 3: Which is which? more precise – Student B
Scalar and Vector Quantities, Random and
Systematic Errors
3A Activity 1: Conversion of Units,
1. SCALAR Scientific Notation, and
2. SCALAR Significant Figures
3. SCALAR
4. VECTOR 1. (a) 3 s.f. (f) 4 s.f.
5. VECTOR (b) 3 s.f. (g) 1 s.f.
6. VECTOR (c) 2 s.f. (h) 2 s.f.
7. SCALAR (d) 4 s.f. (i) 3 s.f.
8. VECTOR (e) 2 s.f. (j) 2 s.f.
9. SCALAR 2. (a) 7.50x104 (c) 1.65x109
10. SCALAR (b) 4.89x10-3 (d) 2.00x106
3. (a) 12 265.2 g (e) 5.50x103
3B (b) 57.61 L (f) 3.11x10-5
1. RANDOM (c) 52.1 kg.m/s2 (g) 1.6x107
2. RANDOM (d) 5.14x10-4 kg/mL(h)3.41x106
3. SYSTEMATIC 4. (a) 5.256x105 km
4. RANDOM (b) 1.075x105 kHz
5. SYSTEMATIC (c) 6.5x10-8 km2
6. SYSTEMATIC (d) 1 m3
7. SYSTEMATIC (e) 1.08x105 m3
8. SYSTEMATIC (f) 3.78x104 mg
9. RANDOM (g) 8.08x102 kg/m3
10. SYSTEMATIC (h) 3.4x10 m/s
Answer Key
APPENDICES

Appendix A. List of metric prefixes, symbols, meaning, and examples

Prefix Symbol Meaning Examples


yotta- Y 1024 1 Ym = 1024 m
zetta- Z 1021 1 Zt = 1021 m
exa- E 1018 1 Em = 1018 m
peta- P 1015 1 Pm = 1015 m
tera- T 1012 1 Tm = 1012 m
giga- G 109 1 Gm = 109 m
mega- M 106 1 Mm = 106 m
kilo- k 103 1 km = 103 m
hecto- h 102 1 hm = 102 m
deka- da 101 1 dam = 101 m
deci- d 10-1 1 dm = 10-1 m
centi- c 10-2 1 cm = 10-2 m
milli- m 10-3 1 mm = 10-3 m
micro- µ 10-6 1 µ m = 10-6 m
nano- n 10-9 1 nm = 10-9 m
pico- p 10-12 1 pm = 10-12 m
femto- f 10-15 1 fm = 10-15 m
atto- a 10-18 1 am = 10-18 m
zepto- z 10-21 1 zm = 10-21 m
yocto- y 10-24 1 ym = 10-24 m

Appendix B. List of conversion factors for length, mass, and time

Length Mass Time


1 m = 100 cm 1 kg = 1 000 g 1 min = 60 s
1m = 1 000 mm 1kg = 0.0685 slug 1 h = 3 600 s
1 km = 1 000 m 1 g = 6.85x10-5 slug 1 day = 86 400 s
1 in = 2.54 cm 1 slug = 14.59 kg 1 y = 3.156x107 s
1 ft = 12 in (1 kg = 2.205 lbm when
1 ft = 30.48 cm g=9.81 m/s2)
1 ft = 0.3048 m 1 lbm = 0.4536 kg
1 yd = 91.44 cm 1 lbm = 453.6 g
1 mi = 1.609 km
1 mi = 5 280 ft

Author: RIC ME D. DIAZ


School/Station: LIANGA NATIONAL COMPREHENSIVE HIGH SCHOOL
Division: SURIGAO DEL SUR
email address: ricme.diaz@deped.gov.ph
WEEKLY LEARNING ACTIVITY SHEETS
General Physics 1, Grade 12, Quarter 1, Week 2

ONE-DIMENSIONAL KINEMATICS

Name: _________________________________________ Section: __________________

Most Essential Learning Competencies:

1. Convert a verbal description of a physical situation involving uniform acceleration


in one dimension into a mathematical description (STEM_GP12Kin-Ib12)
2. Interpret displacement and velocity, respectively, as areas under velocity vs. time
and acceleration vs. time curves (STEM_GP12KINIb-14)
3. Interpret velocity and acceleration, respectively, as slopes of position vs. time and
velocity vs. time curves (STEM_GP12KINIb-15)
4. Construct velocity vs. time and acceleration vs. time graphs, respectively,
corresponding to a given position vs. time-graph and velocity vs. time graph and
vice versa (STEM_GP12KINIb-16)
5. Solve for unknown quantities in equations involving one-dimensional uniformly
accelerated motion, including free fall motion (STEM_GP12KINIb-17)
6. Solve problems involving one-dimensional motion with constant acceleration in
contexts such as, but not limited to, the “tail-gating phenomenon”, pursuit, rocket
launch, and freefall problems (STEM_GP12KINIb-19)

Learning Objectives:
At the end of the lesson, the students should be able to:

1. describe the motion of an object in terms of distance or displacement, speed or


velocity;
2. measure the distance and displacement;
3. solve problems on distance, displacement, speed, velocity and acceleration;
4. describe velocity and acceleration, respectively, as slopes of position vs. time and
velocity vs. time curves;
5. create graphs of velocity vs. time and acceleration vs. time, respectively,
corresponding to a given position vs. time-graph and velocity vs. time graph and vice
versa
6. solve for unknown quantities in equations involving one-dimensional uniformly
accelerated motion, including free fall motion; and
7. solve problems involving one-dimensional motion with constant acceleration.

Time Allotment: 4 hours

Author: AR A. Ranesis
School/Station: Alba Integrated School
Division: Surigao del Sur Division
email address: ar.ranesis@deped.gov.ph
1
Key Concepts

 Frame of reference is an object or point from which movement is determined.


Movement is relative to an object that appears stationary.
 Motion is a change in position relative to a frame of reference.
 Distance (d) is how far an object travels. It does not depend on direction.
Sample problem 1:
What distance did the ant walk?

Figure 1.
Source: https://www.g-pisd.org
Answer: The ant walked 7 cm or d = 7cm.

 Displacement (∆x) is the difference between an object’s final position and its
starting position. It does depend on direction.
Displacement = final position – initial position
∆x = xfinal – xinitial
Sample problem 2:
Find the distance and displacement of the ant.

Figure 2.
Source: https://www.g-pisd.org
Answer: The ant walked 3 cm due east. since displacement is a vector quantity, we
have to consider the direction of the motion. Travelling to the east is positive and
travelling to the west is negative.

Sample problem 3:
A man walks 205 m to the east, then turns back and walks 60 m due west. What
is the total distance he walked? What is his total displacement?

Answer: The total distance he walked is d = 205 m + 60 m = 265 m. However, for


his total or net displacement, we have to consider the directions. Considering the
Cartesian plane, going east is positive and going west is negative. His
displacement is ∆x = 205m east – 60 km west = 145 m, east. Therefore, the net
displacement of the man is 135 m, due east.

Author: AR A. Ranesis
School/Station: Alba Integrated School
Division: Surigao del Sur Division
email address: ar.ranesis@deped.gov.ph
2
 Speed, a scalar quantity refers to the rate of motion. It is the ratio of distance
covered and the time of travel. Like distance, speed does not depend on direction.
In equation,
distance d
Speed  or s 
time t
 Speed, therefore, refers to how fast the object is moving. It tells us that a fast-
moving object has a high speed and covers a large distance in a short period of
time while a slow-moving object that has a low speed covers a relatively small
amount of distance in the same amount of time.
 Since speed is distance divided by time, speed is expressed in terms of the unit of
distance and unit of time. The SI unit for distance is meter, and time is second.
Thus, speed is expressed in meters per second or m/s. For vehicles, the
standard unit for speed is kilometers per hour or km/h (kph). In other countries
such as USA, they use miles per hour or mph as unit of speed. Ship’s speed is
expressed in nautical per hour or knots.
 Average speed is the total distance divided by the total time. It is use to determine
speed which is not constant. In equation,
total distance d
Average speed  or s ave  total
total time of travel t total
n  Speedometer is one of the devices on the instrument panel in front of the driver’s
seat of a motor vehicle that reads the instantaneous speed of the vehicle.

Sample problem 4:
A car drives 100 meters in 5 seconds. What is the car’s average speed?

Solution
s = d/t
s = (100 m)/(5 s)
s = 20 m/s The car’s speed is 20 m/s.

 Velocity is a vector quantity defined as the ratio of the displacement to the total
time elapsed. Velocities in the same direction are combine by adding while
velocities in different directions are combine by subtracting. The average velocity
is the total displacement of an object divided by the elapsed time. In equation,
displaceme nt x
velocity  or v 
total time elapsed t

Sample problem 5:
Usain Bolt is a world known Olympian track and field athlete. He can run the
100-m dash in about 9.58 seconds. What is his velocity?

Solution
v = ∆x/t
v = (100 m)/(9.58 s)
v = 10.44 m/s Usain Bolt’s velocity is 10.44 m/s.

Author: AR A. Ranesis
School/Station: Alba Integrated School
Division: Surigao del Sur Division
email address: ar.ranesis@deped.gov.ph
3
Sample problem 6:
A snake crawls 5 m east, then 3 m west in 20 seconds. What is his distance
traveled, displacement, average speed and average velocity?

Solution:
A. Distance
d=5m+3m
d = 8m

B. Displacement
∆x = 5 m (east) – 3 m (west)
= 2 m due east

C. Average speed
s = (8 m)/(20 s)
= 0.4 m/s

D. Average velocity
v = (2 m)/(20 s) due east
= 0.1 m/s due east

 Acceleration is the rate at which velocity changes. In symbols, average


acceleration ā v v f - v o
a 
t tf - ti
 The SI unit for acceleration is m/s2
 Acceleration is a vector, and thus has a both a magnitude and direction.
 Acceleration can be caused by either a change in the magnitude or the direction of
the velocity.
 Instantaneous acceleration a is the acceleration at a specific instant in time.
 Deceleration is an acceleration with a direction opposite to that of the velocity.
Sample problem 7:
In 2.5 s a car increases its speed from 60 km/h to 67.2 km/h while a
bicycle goes from rest to 9 km/h. Which undergoes the greater acceleration? What
is the acceleration of each vehicle?

Solution:
A. Car’s acceleration
vf = 66 km/h vi = 60 km/h ∆t = 2.5s

∆v = 65 km/h – 7.2 km/h


∆v = 7.2 km/h or 2 m/s

ā = 2 m/s / 2.5 s
ā = 0.8 m/s/s or 0.8 m/s2 The car’s acceleration is 0.8 m/s2
B. Bike’s acceleration
vf = 66 km/h vi = 0 ∆t = 2.5s

Author: AR A. Ranesis
School/Station: Alba Integrated School
Division: Surigao del Sur Division
email address: ar.ranesis@deped.gov.ph
4
∆v = 9 km/h – 0
∆v = 9 km/h or 2.5 m/s

ā = 2.5 m/s / 2.5 s


ā = 1.0 m/s/s or 1.0 m/s2 The bike’s acceleration is 1.0 m/s2

Therefore, The bike has the greater acceleration.

 An object’s motion can be represented by a position-time graph (See Figure 3). In


this graph, the x- axis represents the time and the y- axis represents the position
of the object relative to the starting point. The position-time graph tells us how
far an object has travelled from its starting position at any given time it started
moving.

Figure 3.
Source: https://www.physicsclassroom.com/Class/1DKin/U1L3a2.gif
Sample problem 8
Use Figure 3 to answer the following:
A. At what time is the car’s position equal to 40 m?
Answer: The car is at 40 m at time t = 4 s.

B. What was the car’s position at t = 2 s?


Answer: The car’s position is 20m at t= 2 s.

C. What is the total distance travelled by the car at the end of 5s?
Answer: The car travelled a total distance of 50m.

D. What is the speed of the object from 20 km to 50 km?


∆y⁄ y2 − y1⁄ 50m/s − 20m/s⁄ 30m⁄ = 𝟏𝟎𝐦/𝐬
Answer: m = ∆x = x2 − x1 = 5s − 2s = 3s

Figure 4.
 In Figure 4, the graph shows the object is moving
Source: General
at constant positive velocity. This explains further Physics 1 - Grade
that as the time increases the distance 12 Alternative
also increases with time. Delivery mode,
2020, pdf, page
12

Author: AR A. Ranesis
School/Station: Alba Integrated School
Division: Surigao del Sur Division
email address: ar.ranesis@deped.gov.ph
5
Figure 5.

 In Figure 5, the graph shows object is moving Source: General


Physics 1 - Grade
at constant negative velocity. 12 Alternative
Delivery mode,
2020, pdf, page
12

Figure 6.
Figure 6.
Source: General
 In Figure 6, the graph shows the object is at rest, it Physics 1 - Grade
has zero velocity. 12 Alternative
Delivery mode,
2020, pdf, page
12

Figure 7.
 In Figure 7, the graph shows the object is moving with
Source: General
varying speed. It means that the velocity is
Physics 1 -not
Grade
constant. The object is accelerating. 12 Alternative
Delivery mode,
2020, pdf, page
12

 An object’s motion can be represented by a velocity-time graph (See Figure 8). In


this graph, the x- axis represents the time and the y- axis represents the velocity
of the object relative to the starting point. The velocity-time graph tells us how
fast an object moves. In Figure

Figure 8.
Source: https://www.physicsclassroom.com/Class/1DKin/U1L4a2.gif
Figure 9.
FigureGeneral
Source: 9.
 In Figure 9, the graph shows the object is moving at constant
Physics 1 - Grade
positive acceleration. 12 Alternative
Delivery mode,
2020, pdf, page
13

Author: AR A. Ranesis
School/Station: Alba Integrated School
Division: Surigao del Sur Division
email address: ar.ranesis@deped.gov.ph
6
Figure 10.
 In Figure 10, the graph shows object is moving at
Source: General
constant negative acceleration. Physics 1 - Grade
12 Alternative
Delivery mode,
2020, pdf, page
13

 In Figure 11, the graph shows the object is atFigure 11.


constant velocity. It is not accelerating. Source: General
Physics 1 - Grade
12 Alternative
Delivery mode,
2020, pdf, page
13

 In Figure 12, the graph shows the object is moving


Figure 12.
with varying speed. It means that the velocity is not
Source: General
constant. The object is accelerating. Physics 1 - Grade
12 Alternative
Delivery mode,
2020, pdf, page
13

 The kinematic equations are a set of four equations that can be utilized to predict
unknown information about an object's motion if other information is known. The
equations can be utilized for any motion that can be described as being either a
constant velocity motion (an acceleration of 0 m/s/s) or a constant acceleration
motion.

Equation Variables contained


Equation
Number d vi vf t a

𝑣𝑖 + 𝑣𝑓
1 𝑑=( )𝑡 ✓ ✓ ✓ ✓ ✖
2

2 𝑣𝑓 = 𝑣𝑖 + 𝑎𝑡 ✖ ✓ ✓ ✓ ✓

1
3 𝑑 = 𝑣𝑖 𝑡 + 𝑎𝑡 2 ✓ ✓ ✖ ✓ ✓
2

4 𝑣𝑓 2 = 𝑣𝑖 2 + 2𝑎𝑑 ✓ ✓ ✓ ✖ ✓

Table 1. The Four (4) Kinematic Equations for uniform accelerated motion in horizontal dimension

 Aristotle thought that heavy objects fall faster than light ones, in proportion to
their weight. Galileo Galilei argued that a body should fall downward with an

Author: AR A. Ranesis
School/Station: Alba Integrated School
Division: Surigao del Sur Division
email address: ar.ranesis@deped.gov.ph
7
acceleration that is constant and independent of the body's weight and
composition. This means that heavy or light objects will fall at the same time when
allowed to drop from the same height neglecting air resistance.
 Free Fall is an example of a uniformly accelerated motion. When an object falls
under the influence of gravity alone then it is in a state of free fall.
 Near the surface of the earth, the acceleration due to gravity can be taken as a
constant: g= -9.81 m/s2 (directed downward). At this stage, we shall also neglect
air resistance. Thus, we can take gravity as the only influence on an object in “free
fall”. Problems in "Free Fall" include object thrown upward that reaches a certain
height before falling down. For cases like these, we have to be very careful with
positive and negative signs for the vertical motion.

Equation Number Equation

𝑣𝑖 + 𝑣𝑓
1 𝑦=( )𝑡
2
2 𝑣𝑓 = 𝑣𝑖 + 𝑔𝑡

1
3 𝑦 = 𝑣𝑖 𝑡 + 𝑔𝑡 2
2

4 𝑣𝑓 2 = 𝑣𝑖 2 + 2𝑔y

Table 2 The Four (4) Kinematic Equations for uniform accelerated motion in vertical dimension

 An object dropped from an elevated height


a. initial velocity equal to zero (vi = 0)
b. distance travelled can be calculated using d = ½ gt2
c. displacement can be calculated using d= -½ gt2

 An object thrown vertically downward


a. initial velocity is not equal to zero (vi ≠ 0)
b. final velocity as it reached the ground is vf = vi – gt
c. The distance travelled is calculated using the equation d = vft - ½gt2

 If an object is thrown vertically upward then fall down


a. Initial velocity is not equal to zero (vi ≠ 0)
b. The velocity of its maximum height is equal to zero.
c. The distance travelled in going up is equal to the distance covered in going
down.
d. The time spent in its upward motion is equal to the time spent in its
downward motion, hence, the total time of flight is equal to t = 2t.

Sample problem 8:

Author: AR A. Ranesis
School/Station: Alba Integrated School
Division: Surigao del Sur Division
email address: ar.ranesis@deped.gov.ph
8
A truck slows down from a speed of 35.0 m/s to rest in 7.00 s. How far did
it travel in that time?

Solution:
vi = 35.0 m/s vf = 0 m/s t = 7.00 s d=?

𝑣𝑖 + 𝑣𝑓
𝑑=( )𝑡
2
35 𝑚⁄𝑠 + 0 𝑚⁄𝑠
𝑑=( ) 7.00𝑠
2
𝒅 = 𝟏𝟐𝟐. 𝟓 𝒎
The truck travelled 122.5 m.
Sample problem 9:
A person plays a badminton and hit the shuttlecock upward into the air
with an initial velocity of 25.0 m/s.
a. How high it goes
b. How long the shuttlecock is in the air before it can reach the opponent’s
court.

Solution:
Given: 𝑣𝑖=15.0 𝑚/𝑠

Note that as the ball rises, its speed decreases until it reaches the highest point,
where its speed is zero for an instant, then it descends with increasing speed.

a) To determine the maximum height, we calculate the position of the ball


when its velocity equals zero.

𝑣𝑓2 = 𝑣𝑖2−2𝑔𝑦
𝑦 = 𝑣𝑓2−𝑣𝑖2 / 2𝑎
𝑦 = [0 𝑚/𝑠−(25.0𝑚/𝑠)2 ] / [2(−9.81𝑚/𝑠2)]
𝑦 =31.81 𝑚
The ball reaches a height of 31.81 m.

b) To get the time the ball is in the air, we have

𝑦 = 𝑣𝑖𝑡 − 1/2𝑔𝑡2
0 = (15.0 𝑚/𝑠)𝑡 − 1/2(9.80 𝑚/𝑠2)𝑡2
(15.0 𝑚/𝑠) − 4.90 𝑚/𝑠2𝑡 )𝑡 = 0
𝑡 = 0 ; 𝑡 = (15.0 𝑚/𝑠) / 4.90 𝑚/𝑠2) =3.06 𝑠

The ball is in the air for 3.06 s.

Author: AR A. Ranesis
School/Station: Alba Integrated School
Division: Surigao del Sur Division
email address: ar.ranesis@deped.gov.ph
9
Activity 1. Distance vs Displacement. Speed vs. Velocity. Acceleration.

Objectives:
1. Describe the motion of an object in terms of distance or displacement, speed or
velocity.
2. Measure the distance and displacement.
3. Solve problems on distance, displacement, speed, velocity and acceleration.

What you need: Paper and pen

What to do:
Answer the following problems. Write your solutions on a separate sheet of paper.

1. Bob rides his bicycle on a bike path that is 75 kilometers long to get to his
house that is due east of the bike path. If it takes Bob 15 hours then
a. What is his speed?
b. What is his velocity?

2. Jessica jogs on a path that is 15 kilometers long to get to a park that is south of
the jogging path and another 10 km north before she take a rest. If it takes Jessica
2.5 hours then
a. What is her speed?
b. What is her velocity?

3. A driver starts his parked car and within 5 seconds reaches a speed of 60 km/h,
as he travels east. What is his acceleration?

4. In a summer storm, the wind is blowing with a velocity of 8 m/s north.


Suddenly in 3 seconds, the wind’s velocity is 23 m/s north. What is the wind’s
acceleration?

Activity 2. Graph it!

Objectives:
1. Describe velocity and acceleration, respectively, as slopes of position vs. time
and velocity vs. time curves.
2. Create graphs of velocity vs. time and acceleration vs. time, respectively,
corresponding to a given position vs. time-graph and velocity vs. time graph and
vice versa.

What you need: Paper and pen

What to do:

Author: AR A. Ranesis
School/Station: Alba Integrated School
Division: Surigao del Sur Division
email address: ar.ranesis@deped.gov.ph
10
Construct distance-time graphs from given tabulated data and answer the
following guide questions.

1. Plot the tabulated data (distance versus time, d vs t)


2. Time will be in the x-axis and distance in the y-axis.
3. Connect the points.

Distance Time
d (m) t (s)
0 0
10 2
25 4
60 6
125 8
180 10

1. What is the average velocity of the object?


2. What is the acceleration of the object between t = 2 s to t = 6 s?
3. Does the graph show the object is moving with varying or constant speed?
4. Based on your graph, is the object’s acceleration is constant, decelerating, or
accelerating? Why?

Rubric for Guide Questions 3 and 4


3 2 1
Explanations are Explanations are Explanations are
conceptually correct conceptually correct and conceptually incorrect.
and complete. incomplete.

Activity 3. Solve the unknown!


Objective: Discuss how the phenomenon of dispersion relates to Snell’s Law.

What you need: Paper and pen

What to do: Answer the following problems. Write your solutions on a separate sheet of
paper.

1. An airplane accelerated uniformly from rest at the rate of 6.25 m/s2 south for
15 s. What final velocity did it attain?

2. John is driving his sports car down a four-lane highway at 60 m/s. He


overtakes a slow-moving dump truck. If he can accelerate at 7.5 m/s2, how long
will it take for him to reach the speed of 100 m/s?

Author: AR A. Ranesis
School/Station: Alba Integrated School
Division: Surigao del Sur Division
email address: ar.ranesis@deped.gov.ph
11
3. Suppose that a ball is dropped from a tower 70.0 m high. How far will it fall
after 3.00 s?

4. John throws a ball upward into the air with an initial velocity of 15.0 m/s.
a. how high it goes, and
b. how long is the ball in the air before it comes back to his hand.

Reflection

Speedometer, instrument that indicates the speed of a vehicle, usually combined


with a device known as an odometer that records the distance traveled. The speedometer
and odometer are useful features in all vehicles, helping drivers to measure their pace
and estimate the distances they travel. They give important information to the drivers,
helping them to make decisions that can affect their travel.
If speedometers and odometers were not invented, what do you think is its impact to
our road traffic today? Write your 5-sentence answer in a separate sheet of paper.

Rubric for Reflection


3 2 1
Practical application is Practical application is Practical application
scientifically explained scientifically explained is explained consistent to
consistent to the concepts, consistent to the concepts, the
and has no misconceptions. but with minimal concepts but with
misconceptions. misconceptions.

References for learners:

Lingatong, Leah Lyn A. General Physics 1 Grade 12 Alternative Delivery Mode. 1st ed,
Department of Education, 2020

Lumen Boundless Physics. “Acceleration”


Accessed on August 20, 2021
https://courses.lumenlearning.com/physics/chapter/2-4-acceleration/

Gregory-Portland Independent School District. Distance, Displacement, Speed, and


Velocity
Accessed on August 19, 2021
https://www.g-pisd.org

The Physics Classroom. “Kinematic Equations and Problem-Solving”


Accessed on August 23, 2021
https://www.physicsclassroom.com/Class/1DKin/U1L6b.cfm

The Physics Classroom. “The Meaning of Shape for a p-t Graph”


Accessed on August 20, 2021

Author: AR A. Ranesis
School/Station: Alba Integrated School
Division: Surigao del Sur Division
email address: ar.ranesis@deped.gov.ph
12
https://www.physicsclassroom.com/class/1DKin/Lesson-3/The-Meaning-of-
Shape-for-a-p-t-Graph

The Physics Classroom. “The Meaning of Shape for a v-t Graph”


Accessed on August 20, 2021
https://www.physicsclassroom.com/class/1DKin/Lesson-4/Meaning-of-Shape-f
or-a-v-t-Graph

Answers Key

4. 5 m/s2
3. 3.33 m/s2
b) 2 km/h due south 2. a) 10 km/h
b) 5 km/h due east 1. a) 5 km/h
Activity 1

Time (s)

Distance (m)

the speed is varying and the line in the graph is curve upwards.
4. Based on the graph, it can be interpreted that the object is accelerating because
3. The graph shows that the speed is varying.
2. 1,25 m/s2
1. 18 m/s
Activity 2

b) t= 3.06 s 4. a) y =11.5 m
3. y = 1.44 m
2. 5.33 s
1. 92.75 m/s
Activity 3
Author: AR A. Ranesis
School/Station: Alba Integrated School
Division: Surigao del Sur Division
email address: ar.ranesis@deped.gov.ph
13
WEEKLY LEARNING ACTIVITY SHEETS
General Physics I 12 Quarter 1 Week 3

KINEMATICS: MOTION IN 2- DIMENSIONS AND 3-DIMENSIONS

Name: ___________________________________________ Section: ____________________

Most Essential Learning Competencies


 Describe motion using the concept of relative velocities in 1D and 2D. (STEM_GP12KIN-
Ic20)
 Deduce the consequences of the independence of vertical and horizontal components of
projectile motion. (STEM_GP12KIN-Ic22)
 Calculate range, time of flight, and maximum heights of projectiles. (STEM_GP12KIN-Ic23)
 Solve problems involving two-dimensional motion in contexts such as, but not limited to
ledge jumping, movie stunts, basketball, safe locations during firework displays, and
Ferris wheels. (STEM_GP12KIN-Ic26)

Specific Objectives
1. Define relative velocity
2. Solve problems involving relative velocities.
3. Describe the vertical and horizontal component of object’s motion when undergoing
projectile motion.
4. Calculate range, time of flight and maximum heights of projectiles
5. Solve problems involving two-dimensional motion in contexts such as, but not
limited to ledge jumping, movie stunts, basketball, safe locations during firework
displays

Time Allotment: 4 hours

Key Concepts

Relative Motion
 The idea of relative motion can be explained
through a boat crossing a river. The wind can
influence the movement of a boat moving from
one to the opposite side of the river. The
movement will be affected by the river’s flow. This
implies that the boat will not reach the shore
directly across from the point where it started.
The boat will be carried downstream. Although
speedometer of the boat will read a certain value
of its speed, the observer on the other side of the
shore will be reading greater than its value. Figure 1: Boat moving relative to the flow
of current in the river
 Relative velocity is defined as the velocity of an
object B in the rest frame of another object A. Source: https://courses.lumenlearning.com/suny-
osuniversityphysics/chapter/4-5-relative-motion-
in-one-and-two-dimensions/

Author: Marvelous Saint P. Jumanoy


School/Station: GAMUT NATIONAL HIGH SCHOOL - Tago
Division: SURIGAO DEL SUR
email address: marveloussaint.jumanoy@deped.gov.ph
 Supposing a man walking on above a long flat train both moving to the right path. In
any time interval, the displacement of the man relative to the Earth is just the sum of
displacement relative to the train and their (man and train) displacement relative to the
Earth.
Hence,

𝑋𝑚an/𝐸𝑎𝑟𝑡ℎ = 𝑋𝑚an/train + 𝑋train/𝐸𝑎𝑟𝑡ℎ


𝑑𝑥 𝑚/𝐸 𝑑𝑡 = 𝑑𝑋 𝑚/t 𝑑𝑡 + 𝑑𝑋 t/𝐸 𝑑𝑡, thus:
𝒗𝒎/𝑬 = 𝒗𝒎/t + 𝒗t/𝑬

𝒗 = 1.0 m/s
𝒗 = 30.0 m/s

Figure 2: man moving relative to Earth and train


 m/E means velocity of motorcycle with respect to Earth. In writing this equation, make
sure that the first subscript on the left side of the equation is the first subscript on the
first term of the equation while the second subscript at the right side of the equation is
the found at the right side of the second term in the equation.

Example 1. Refer figure above, If the man is moving at 1 m/s on the train and the train
at 30 m/s to the right, then the velocity m/E (man with respect to Earth) is moving at
43 m/s to the right. Calculated,
𝑣m/𝐸 = 𝑣m/t + 𝑣t/𝐸
𝑣m/𝐸 = 1m/s + (30 m/s)
= 31 m/s

*If the man is moving at same velocity but this time to the left, the velocity m/E (man
with respect to Earth) is moving at 29 m/s to the left. It is just the algebraic sum of the
velocities.

Example 2: In figure 1, a boat heads north in still water at 4.5 ⃗ 𝑩/𝑬


𝒗
m/s directly across a river that is running east at 3.0 m/s.
What is the velocity of the boat with respect to Earth?
⃗ 𝑩/𝑹
𝒗

Solution: Labeling subscripts for the vector equation, we have


B = boat, R = river, and E = Earth. The vector equation ⃗ 𝑹/𝑬
𝒗
becomes
Figure 3: vector using right
⃗ 𝑩/𝑬 = 𝒗
𝒗 ⃗ 𝑩/𝑹 + 𝒗⃗ 𝑹/𝑬 triangles
⃗ 𝑩/𝑬 = 𝒗
𝒗 ⃗ 𝑩/𝑹 + 𝒗
⃗ 𝑹/𝑬 Source: Young, H. and Freedman, R.
We have right triangle geometry shown in Figure 3. (2016). University Physics with Modern
Physics. Pearson

Author: Marvelous Saint P. Jumanoy


School/Station: GAMUT NATIONAL HIGH SCHOOL - Tago
Division: SURIGAO DEL SUR
email address: marveloussaint.jumanoy@deped.gov.ph
Solving for ⃗ 𝑩/𝑬 , we have
𝒗
⃗ 𝑩/𝑬 =
𝒗 √𝒗
⃗⃗ 𝑩/𝑹 2 + ⃗𝒗
⃗ 𝑹/𝑬 2 = √(4.5)2 + (3.0)2

= √(4.5)2 + (3.0)2

⃗ 𝑩/𝑬
𝒗 = 5.4m/s,

θ=tan−1(3.04.5) = 33.7°.

Projectile Motion
 Projectile motion is a form of motion where an object moves in a bilaterally
symmetrical, parabolic path. The path that the object follows is called its trajectory.
Projectile motion only occurs when there is one force applied at the beginning on the
trajectory, after which the only interference is from gravity.
 The concept of projectile motion is widely
used in various sports. In playing soccer,
when you kick the ball it follows a path
similar to a parabola. As the ball reaches
the highest peak, it goes back to the
ground as influenced by the gravity and
air resistance.
 Objects that are projected from, and land
on the same horizontal surface will have a Figure 4: Ballistic pathway of the ball
vertically symmetrical path.

 The time it takes from an object to be projected and land is called the time of flight.
This depends on the initial velocity of the projectile and the angle of projection.

 When the projectile reaches a vertical velocity of zero, this is the maximum height of
the projectile and then gravity will take over and accelerate the object downward.

 The horizontal displacement of the projectile is called the range of the projectile, and
depends on the initial velocity of the object

 Forces involved on a projectile are drag, object’s weight, wind and lift due to spinning
motion.

 Objects with reasonable mass moves with lower speed. Objects with higher speed and
forces becomes a significant factor, the idealized model for projectile model could not
anymore fit. This is also true for lighter objects due to influence of external forces which
could affect the movement of the ball. Therefore, idealized model considers weight as the
only force.

Author: Marvelous Saint P. Jumanoy


School/Station: GAMUT NATIONAL HIGH SCHOOL - Tago
Division: SURIGAO DEL SUR
email address: marveloussaint.jumanoy@deped.gov.ph
 The projectile motion is a combination of
horizontal and vertical components of motions
with constant acceleration. These are
independent from each other and we will
analyze them separately. These motions are
just superimposed from each other. The figure
4 shows the direction of velocity components
along x and y-axis.

Along x –axis
𝒗𝒙=𝒗𝟎𝒙
𝒙= 𝒙𝒐+𝒗𝟎𝒙𝒕 (1)

Along y –axis; where a = -g


𝒗𝒚=𝒗𝟎𝒚− 𝒈𝒕
1
𝒚=𝒚𝟎+𝒗𝟎𝒚𝒕− 2𝒈𝒕𝟐 (2)
Figure 5: the direction of velocity components along
x and y axis

 Since velocity along x-component is constant, the acceleration is zero. On the


other hand, velocity along y-component is moving at constant gravitational
acceleration. The value of the gravitational acceleration (g) is equal to 9.8 m/s2.
 The figure below shows another case of projectile motion thrown at some angle.
The projectile trajectory is a parabola. The velocity along y component decreases
and becomes zero when it reaches the highest peak. When it goes back to Earth’s
ground, the velocity along y component increases but directed in opposite
direction. The velocity along x component, on the other hand, is constant
throughout the path.

Along x-axis
𝑣𝑥=𝑣0𝑥
𝑤ℎ𝑒𝑟𝑒 𝑣0𝑥= 𝑣0𝑐𝑜𝑠𝜃
Then, 𝒗𝒙=𝒗𝟎𝒄𝒐𝒔𝜽 (3)

Along y-axis
𝑣𝑦=𝑣0𝑦 -𝑔𝑡
Where 𝑣0𝑦= 𝑣0𝑠𝑖𝑛𝜃 Figure 6. ball in a projectile motion leaves its trajectory
Then, 𝒗𝒚=𝒗𝟎𝒔𝒊𝒏𝜽− 𝒈𝒕 (4) path
 Similarly, in the vector resolution lesson, you need to analyze the vector using
right triangles.

 From equation 2, we substitute 𝑣0𝑥;


𝑥 = 𝑥𝑜 + 𝑣0𝑥𝑡
Where, 𝑣0𝑥 = 𝑣0𝑐𝑜𝑠𝜃
𝒙 = 𝒙𝒐 + 𝒗𝟎 𝒄𝒐𝒔𝜽 𝒕 (5)

 From equation 4, we substitute 𝒗𝟎𝒚;

Author: Marvelous Saint P. Jumanoy


School/Station: GAMUT NATIONAL HIGH SCHOOL - Tago
Division: SURIGAO DEL SUR
email address: marveloussaint.jumanoy@deped.gov.ph
1
𝑦 = 𝑦0 + 𝑣0𝑦𝑡 − 2 𝑔𝑡2
Where 𝑣0𝑦 = 𝑣0𝑠𝑖𝑛𝜃
1
𝒚 = 𝒚𝟎 + 𝒗𝟎 𝒔𝒊𝒏𝜽 𝒕 − 2 𝒈𝒕 𝟐 (6)

 distance r of the projectile;

r = √𝒙𝟐 + 𝒚𝟐 (7)

 The speed of a projectile at any given time;

𝒗=√𝒗𝒙𝟐+𝒗𝒚𝟐 (8)

 The direction of projectile in terms of angle;


vy
𝒕𝒂𝒏 Ɵ = (9)
vx

Figure 7: vector of gravitational acceleration


 the observation for acceleration vectors can then be shown using the figure above.
Since velocity along x-component is constant, the acceleration is zero. On the other
hand, velocity along y-component is moving at constant acceleration. The value of
acceleration is equal to 9.8 m/s2. The equations can also prove that we are dealing
with a parabola.

Author: Marvelous Saint P. Jumanoy


School/Station: GAMUT NATIONAL HIGH SCHOOL - Tago
Division: SURIGAO DEL SUR
email address: marveloussaint.jumanoy@deped.gov.ph
follows the general equation for parabola 𝑦 = 𝑎𝑥 2 + 𝑏

Take note that the angle Ɵ can produce various parabolic trajectory when launched
at speed 𝒗𝟎.

Given these assumptions, the following steps are then used to analyze projectile
motion:

(Consider the Figure 8)

Step 1. Resolve or break the motion into horizontal


and vertical components along the x- and y-
axes. These axes are perpendicular, so Ax = A
cos θ and Ay = A sin θ are used. The magnitude of
the components of displacement s along these
axes are x and y. The magnitudes of the
components of the velocity v are Vx = V
cos θ and Vy = v sin θ where v is the magnitude of Figure 8: A soccer player kicks the ball and ball
the velocity and θ is its direction, as shown in 2. leaves its trajectory path
Initial values are denoted with a subscript 0, as source:https://courses.lumenlearning.com/physi
usual. cs/chapter/3-4-projectile-motion/

Step 2. Treat the motion as two independent one-


dimensional motions, one horizontal and the other vertical. The kinematic equations for
horizontal and vertical motion take the following forms:

Horizontal Motion (ax = 0)

x = x0 + vxt

vx = v0x = vx = velocity is a constant.

Vertical Motion (assuming positive is up ay = -g = -9.8 m/s2)


1
𝑦 = 𝑦0 + (𝑣0𝑦+ 𝒗𝒚 ) 𝑡
2

𝒗𝒚 = 𝑣0𝑦 −gt

y=y0+v0yt−21 gt2

vy2=v0y2−2g(y−y0)
Step 3. Solve for the unknowns in the two separate motions—one horizontal and one
vertical. Note that the only common variable between the motions is time t. The
problem-solving procedures here are the same as for one-
dimensional kinematics and are illustrated in the solved examples below.
Step 4. Recombine the two motions to find the total displacement s and velocity v.
Because the x – and y -motions are perpendicular and employing;

A=√A𝑥 2 + A𝑦 2 ;

Author: Marvelous Saint P. Jumanoy


School/Station: GAMUT NATIONAL HIGH SCHOOL - Tago
Division: SURIGAO DEL SUR
email address: marveloussaint.jumanoy@deped.gov.ph
𝐴𝑦
and θ = tan−1 ( ) in the following form, where θ is the direction of the
𝐴𝑥
displacement s and θv is the direction of the velocity v:
Example 1: During a fireworks display, a shell is shot
into the air with an initial speed of 70.0 m/s at an
angle of 75.0º above the horizontal, as illustrated in
Figure 3. The fuse is timed to ignite the shell just as
it reaches its highest point above the ground.

(a) Calculate the height at which the shell explodes.

(b) How much time passed between the launch of the


shell and the explosion?

(c) What is the horizontal displacement of the shell


when it explodes?

Strategy:

Because air resistance is negligible for the


Figure 9: The trajectory of a fireworks shell. The fuse is
unexploded shell, the analysis method outlined
set to explode the shell at the highest point in its
above can be used. The motion can be broken into trajectory, which is found to be at a height of 233 m and
horizontal and vertical motions in which ax = 0 125 m away horizontally.
and ay = –g. We can then define x0 and y0 to be
zero and solve for the desired quantities. Source:
https://courses.lumenlearning.com/physics/chapter/3-4-
projectile-motion/
Solution for (a):

By “height” we mean the altitude or vertical position y above the starting point. The
highest point in any trajectory, called the apex, is reached when vy=0. Since we know the
initial and final velocities as well as the initial position, we use the following equation to
find y:

Because y0 and vy are both zero, the equation simplifies to

0 = vOy2 – 2gy.

Solving for y gives

y=v0y2/2g.

Now we must find v0y, the component of the initial velocity in the y-direction. It is given
by v0y = v0 sin θ, where v0y is the initial velocity of 70.0 m/s, and θ0 = 75.0º is the initial
angle. Thus,

vOy = v0 sin θ0 = (70.0 m/s)(sin 75º)

vOy = 67.6 m/s, and y = 233 m

Solution for (b):

As in many physics problems, there is more than one way to solve for the time to the
1
highest point. In this case, the easiest method is to use 𝑦 = 𝑦0 + 2 (𝑣0𝑦+ 𝒗𝒚 ) 𝑡. Because y0 is
zero, this equation reduces to simply,

Author: Marvelous Saint P. Jumanoy


School/Station: GAMUT NATIONAL HIGH SCHOOL - Tago
Division: SURIGAO DEL SUR
email address: marveloussaint.jumanoy@deped.gov.ph
1
𝑦= (𝑣0𝑦+ 𝒗𝒚 ) 𝑡
2

Note that the final vertical velocity, vy, at the highest point is zero. Thus,
2y 2(233 m
t= (v0y+vy)
= (67.6 m/s) = 6.90 s

Solution for (c)

Because air resistance is negligible, ax=0 and the horizontal velocity is constant, as
discussed above. The horizontal displacement is horizontal velocity multiplied by time as
given by x = x0 + vxt, where x0 is equal to zero:

x = vx t,

where vx is the x-component of the velocity, which is given by vx = v0 cos θ0,

Now,

vx = v0 cos θ0 = (70.0 m/s)(cos 75º) = 18.1 m/s

The time t for both motions is the same, and so x is

x = (18.1 m/s)(6.90 s) = 125 m.


Activity 1: Relative Motion Short Quiz
Objectives
 Define relative velocity
 Solve problems involving relative velocities.
What you need: pencil/ballpen, sheets of paper, scientific calculator

What to do:
Choose the letter of the best answer. Write the chosen letter on a separate sheet of paper.
1. What is best describes a relative velocity?
a. the velocity of an object in the rest frame of another object
b. the acceleration of an object with respect to earth
c. the motion of object towards another moving object
d. the speed of object with respect to acceleration of another object
2. Which is true about relative motion?
a. All motions are at rest at all reference
b. All motions are relative to some frame of reference
c. Relative motion is relative to static universe
d. Relative motion does not apply in any reference

3. Find the truck’s velocity relative to you when you drive north on a straight-two lane road at
a constant 88 km/h. A truck in another lane approaches you at a constant 104 km/h.
a. −192 𝑘𝑚/ℎ
b. 192 km/h
c. -200 km/h
d. 185 km/h
4. A truck is traveling south at a speed of 70 km/h toward an intersection. A car is traveling
east toward the intersection at a speed of 80 km/h. What is the velocity of the car relative
to the truck?
a. 110.0 km/h

Author: Marvelous Saint P. Jumanoy


School/Station: GAMUT NATIONAL HIGH SCHOOL - Tago
Division: SURIGAO DEL SUR
email address: marveloussaint.jumanoy@deped.gov.ph
b. 103.0 km/h
c. 160.0 km/h
d. 106.0 km/h
5. From the item number four (4) problem, find the angle θ and direction?
a. 40.2°north of east
b. 41.2°north of east
c. 45.2°north of east
d. 51.2°north of east

Activity 2: Projectile Motion

Objectives:
 Describe the vertical and horizontal component of object’s motion when undergoing
projectile motion.
 Calculate range, time of flight and maximum heights of projectiles
 Solve problems involving two-dimensional motion in contexts such as, but not limited to
ledge jumping, movie stunts, basketball, safe locations during firework displays

What you need


 pencil/ballpen
 sheets of paper
 scientific calculator

What to do
A. Read and answer as directed. Write your answers in a separate sheet of paper.
1.What is the projectile’s horizontal accelerations when it was thrown at an angle of 30
degrees above its horizontal?
a. zero b.9.81 m/s2
c.it varies d. insufficient information

2.A player kicks the ball with a velocity of 25 m/s directed 53 degrees above the
horizontal. What is the vertical component of its initial velocity?
a.15 m/s b.20 m/s c.33 m/s d.25 m/s

3.At what other angle will the football be kicked to travel 50 yards if its initial velocity was
the same with the ball kicked at 25 degree and travels 50 yards?
a.90 degrees b.45 degrees c.55 degrees d.65 degrees

4.Two balls were thrown horizontally from the same height. Ball A has speed of 0.4 m/s
while ball B has a speed of 20 m/s. The time takes for Ball B to reach the ground
compared to Ball A is
a. same b. twice c. half d. four times

5. The ball was fired initially at 12 m/s from a cannon facing northwards. The cannon
moves eastward at 24 m/s. Which of the vectors represent the resultant velocity of the
ball?

Author: Marvelous Saint P. Jumanoy


School/Station: GAMUT NATIONAL HIGH SCHOOL - Tago
Division: SURIGAO DEL SUR
email address: marveloussaint.jumanoy@deped.gov.ph
6. An arrow was thrown at angle of 45 degrees while the other arrow was thrown at 60
degrees. Compared to arrows fired at 60 degrees, the arrow fired at 45 degrees is ______.
a. longer time of flight and range c. longer time of flight and shorter range
b. shorter time of flight and longer range d. shorter time of flight and range

B. Solve the following problems legibly. Show your solution and it in separate sheet of
paper.

7. Kilauea in Hawaii is the world’s most continuously active volcano. Very active volcanoes
characteristically eject red-hot rocks and lava rather than smoke and ash. Suppose a
large rock is ejected from the volcano with a speed of 25.0 m/s and at an angle 35.0º
above the horizontal, as shown in Figure below. The rock strikes the side of the volcano at
an altitude 20.0 m lower than its starting point. (a) Calculate the time it takes the rock to
follow this path.

Source: https://courses.lumenlearning.com/physics/chapter/3-4-projectile-motion/

8. A bicycle stunt rider rides off the edge of a cliff. Just as the edge of its velocity is
horizontal with magnitude of 9.0 m/s. Find the position, distance from the edge of cliff at
0.5 s after it leaves the edge of the cliff?

9. A batter hits a baseball so that it leaves the bat at a speed of 𝑣0 = 37.0 𝑚/𝑠 at an angle of
53.1°. Find the position of the ball and its velocity at t = 2 s?

10. Refer from the problem item number 9, Find the time when the ball reaches the highest
point?

Reflection: In a separate sheet of paper, write a five-sentence explanation how relative motions
and projectile motions are involved in the field of sports?
Rubrics
5 4 3 2 0
Practical application Practical application Practical application Practical No discussion at all.
is scientifically is scientifically is explained applications are
explained consistent explained consistent consistent to the explained consistent
to the concepts, and to the concepts, but concepts but with to the concepts but
has no with minimal one or two with more than two
misconceptions. misconceptions. misconceptions. misconceptions.

Author: Marvelous Saint P. Jumanoy


School/Station: GAMUT NATIONAL HIGH SCHOOL - Tago
Division: SURIGAO DEL SUR
email address: marveloussaint.jumanoy@deped.gov.ph
References for learners:

Department of Education. (2018). General Physics 1 Reader.

Young, H. and Freedman, R. (2016). University Physics with Modern Physics. Pearson

Sears, F., Zemansky, M. and Young H. (1992). College Physics 7th Edition. AddisonWesley
Publishing Company

Links: https://courses.lumenlearning.com/physics/chapter/3-1-kinematics-in-two-dimensions-
an-introduction/

https://www.physicsclassroom.com/class/vectors/Lesson-2/Initial-Velocity-Components

Answer Key

B 5.
D 4. highest peak at t = 3.02 s
A 3. 10. the ball reaches the
B 2. m, 39.6m) at t = 2s.
A 1. 9. the ball is located at (44.4
0.5s
Short Quiz (4.5 m, -1.225 m) at t =
Activity 1: Relative Velocity 8. The stunt rider is located
7. t = 3.96 s
6. A
5. C
4. A
(Answer may vary)
3. C
2. A
Reflection
1. A

Activity 2: Projectile Motion

Author: Marvelous Saint P. Jumanoy


School/Station: GAMUT NATIONAL HIGH SCHOOL - Tago
Division: SURIGAO DEL SUR
email address: marveloussaint.jumanoy@deped.gov.ph
WEEKLY LEARNING ACTIVITY SHEETS
General Physics 1, Grade 12, Quarter 1, Week 4

NEWTON’S FIRST LAW OF MOTION

Name: ___________________________________________ Section: ____________________

Learning Competencies:
- Define inertial frames of reference (STEM_GP12N-Id-28)
- Identify action-reaction pairs (STEM_GP12N-Id-31)
- Draw free-body diagrams (STEM_GP12N-Id-32)
- Apply Newton’s 1st law to obtain quantitative and qualitative conclusions about the
contact and noncontact forces acting on a body in equilibrium (STEM_GP12N-Ie-33)
- Differentiate the properties of static friction and kinetic friction (STEM_GP12N-Ie-34)

Specific Objectives:
After the lesson, the students will be able to:
1. Draw free-body diagrams on the forces acting on the objects.
2. Perform simple experiment applying newton’s law of motion.
3. Classify daily activities into static and kinetic friction.

Time Allotment: 4 hours

Key Concepts
 Sir Isaac Newton has significant contribution in the field of Physics as he combined his
idea with other scientists like Galileo, who have the most unified picture of how the
universe work. Newton formulated the three laws of motion and gravitation by which we
can be able to predict the movement of everything around us.
 When no net force acts on a body, the body either remains at rest or moves with constant
velocity in a straight line. Once a body has been set in motion, no net force is needed to
keep it moving. This is known as Newton’s First Law of Motion or Law of Inertia. A
body at rest continues to remain at rest and a body in constant velocity continues to be in
constant uniform motion, unless acted upon by an unbalanced force.
 Newton’s first law tells that when a body is acted on by zero net force, it moves with
constant velocity and zero acceleration. But when the net force is not zero, the inertial
properties of a body are characterized by its mass. The acceleration of a body under the
action of a given set of forces is directly proportional to the vector sum of the forces (the net
force) and inversely proportional to the mass of the body. This relationship is called as
Newton’s Second Law of Motion or Law of Acceleration.
 Like Newton’s first law, this law is valid only in inertial frames of reference. The unit of
force is defined in terms of the units of mass and acceleration. In SI units, the unit of
force is newton (N) which is equal to 1 kg ꞏ m/s2. In symbols,
Fnet = ma
 Newton’s Third Law of Motion or Law of Interaction states that when two bodies
interact, they exert forces on each other that at each instant are equal in magnitude and
opposite in direction. These forces are called action and reaction forces. Each of these
forces acts on only one of the two bodies; they never act on the same body. Newton’s
Third Law of Motion simply states that
For every action, there is an equal and opposite reaction.

Author: Shekaina Faith C. Lozada


School/Station: Gamut National High School
Division: Surigao del Sur
email address: shekainah.lozada@deped.gov.ph
 The statement means that in every interaction, there is a pair of forces acting on the two
interacting objects. The size of the forces on the first object equals the size of the force on
the second object. The direction of the force on the first object is opposite to the direction
of the force on the second object. Forces always come in pairs - equal and opposite action-
reaction force pairs.
 Inertia is the property of matter in which an object that is at rest wants to remain at
rest, and an object that is moving wants to remain moving in a straight line unless
another force acts upon it. Likewise, an inertial frame of reference is a reference frame
in which an object stays either at rest or at a constant velocity unless another force acts
upon it.
 Free-body diagrams are diagrams used to show the relative magnitude and direction of
all forces acting upon an object in a given situation. It is a physical model of an object
that is affected by several forces, both contact and non-contact forces. It is a diagram
showing the chosen body by itself with vectors drawn to show the magnitudes and
direction of all the forces applied on the body by various other bodies interact with it.
 Moreover, it is usually represented by an arrow. The size of the arrow in a free-body
diagram reflects the magnitude of the force. The direction of the arrow shows the direction
that the force is acting. Each force arrow in the diagram is labelled to indicate the exact
type of force.
 It is generally customary in a free-body diagram to represent the object by a box and to
draw the force arrow from the center of the box outward in the direction that the force is
acting. An example of a free-body diagram is shown below.

Fig. 1 Free-body Diagram


Source: General Physics 1 for Senior High School, 1st Edition
 The free-body diagram above depicts four forces acting upon the object. Objects do not
necessarily always have four forces acting upon them. There will be cases in which the
number of forces depicted by a free-body diagram will be one, two, or three. There is no
hard and fast rule about the number of forces that must be drawn in a free-body diagram.
The only rule for drawing free-body diagrams is to depict all the forces that exist for that
object in the given situation.
 STEPS IN CONSTRUCTING FREE – BODY DIAGRAMS
1. Identify which forces are present.
2. Determine the direction in which each force is acting.
3. Draw a box and add arrows for each existing force in the appropriate direction; label
each force arrow according to its type.

Author: Shekaina Faith C. Lozada


School/Station: Gamut National High School
Division: Surigao del Sur
email address: shekainah.lozada@deped.gov.ph
Sample Situations:
1. 2.

Fig. 2 Free-body Diagram of a runner Fig. 3 Free-body Diagram of basketball player


Source: “Sears and Zemansky’s University Source: “Sears and Zemansky’s University
Physics with Modern Physics”, 12th edition. Physics with Modern Physics”, 12th edition.

3.

Fig. 4 Free-body Diagram of a swimmer


Source: “Sears and Zemansky’s University
Physics with Modern Physics”, 12th edition.

 The Law of Inertia or Newton’s first law of motion consider only equilibrium of a body that
can be modelled as a particle. This means that when an object is at rest or moving with
constant velocity in an inertial frame of reference, the net force on it which is the vector
sum of all the forces acting on it must be zero:
ƩF = 0
 This law implies that there is no fundamental difference between bodies at rest and one
that is in constant motion.
 Forces may act on the body and produce no motion. Thus, when two bodies interact by
direct contact of their surfaces, the interaction is being described in terms of contact
forces. One example of contact force is Friction.
 Friction is a force that acts to oppose sliding motion between surfaces. When a body
rests or slides on a surface, the surface exerts a single contact force on the body, with
force components perpendicular and parallel to the surface. The perpendicular
component vector is the normal force, denoted by FN. The component vector parallel to
the surface is the friction force, denoted by Ff. If the surface is frictionless, then f is zero
but there is still a normal force. The direction of the friction force is always such as to
oppose relative motion of the two surfaces.
 The kind of friction that acts when a body slides over a surface is called kinetic friction
force, fk while static friction force act when there is no relative motion. In symbols,
frictional force Ff is
Ff = µFN

Author: Shekaina Faith C. Lozada


School/Station: Gamut National High School
Division: Surigao del Sur
email address: shekainah.lozada@deped.gov.ph
where, µ is the coefficient of friction (static or kinetic) which depends on the nature of
surface and are usually less than one.
 Friction depends upon the surfaces in contact. However, it is also assume that friction
does not depend on the area of the surfaces nor the speed of the relative motion of the
objects. The magnitude of the frictional force is proportional to the magnitude of the
force pushing one surface against the other.
Sample Problem:
A cardboard box weighing 200 N is resting on a wooden tabletop. If the coefficient
of friction between the box and the table top is 0.25, how much force would it take to
keep the box sliding along at constant speed?

Given:
FN = 200 N
µ = 0.25
Solution:
Ff = µFN
Ff = (0.25) (200 N)
= 50 N

Activity 1: Draw Me!

Objective: Draw free-body diagrams on the forces acting on the objects.

What you need:


 Paper and pen

What to do 1: Draw a free-body diagram for each problems. Write your answers in a separate
sheet of paper.
1. A rightward force is applied to a book in order to move it across a desk with a rightward
acceleration. Consider frictional forces. Neglect air resistance.
2. An iceboat is at rest on a perfectly frictionless horizontal surface. A wind is blowing
(along the direction of the runners) after the iceboat is released.
3. A sailboat is at rest when anchored at the wharf. The forces acting on it are the weight of
the boat pulling it down and the buoyant force of the water that is pushing it up. The two
forces acting on the sailboat are equal in magnitude but oppositely directed. Thus, the
vector sum of the forces is zero. Draw the free-body diagram of the situation (at rest and
in motion).
What to do 2: Identify at least three pairs of action-reaction pairs in the following diagram.

Activity 2: Perform It!

Objective: Perform simple experiment applying newton’s law of motion.

What you need:


 Paper and pen
 Boiled Egg

Author: Shekaina Faith C. Lozada


School/Station: Gamut National High School
Division: Surigao del Sur
email address: shekainah.lozada@deped.gov.ph
 Raw Egg
 Marble
 Balloon
 Clip
 String

What to do: Perform the following activities. Write your observations and answers on a separate
sheet of paper.

Activity 2.1 : Spin Me


1. Prepare a hard-boiled egg and a raw egg.
2. Place a hard-boiled egg on its side and spin it.
3. Put your finger on it gently while it is still spinning in order to stop it.
4. Place the raw egg on its side and spin it. Place your finger gently on the egg until it
stops.
Answer the following questions:
a. What do you observe when you gently put your finger on the hard-boiled egg
while it is still spinning?
b. What do you observe when you gently put your finger on the raw egg while it
is still spinning?
c. Does your observation on the spinning of two eggs the same? If no, how does
it happen?
d. What do you think this activity demonstrate?

Activity 2.2 : The Greater


1. Drop a marble and a wadded-up piece of paper at the same time.
2. Push the 2 same objects at the same time with the same force.
3. Push one object harder than the other.
Answer the following questions:
a. What do you observe when you drop a marble and a wadded-up piece of paper
at the same time?
b. What do you observe when you push the 2 same objects at the same time with
the same force?
c. Does your observation change when you push one object harder than the
other? How does it happen?
d. What do you think this activity demonstrate?

Activity 2.3 : Cupids Arrow Balloon


1. Blow up a balloon. Secure the end with a clip so no air escapes.
2. Decorate your balloon to look like an arrow.
3. Set up your anchor points about 10 feet apart. Identify the starting and ending point.
4. Thread your string through a straw, then secure the string to the two anchor points.
Attach your balloon to the straw using tape.
5. Release the balloon to start it racing. Record your observation.
6. Pull the balloon back to the starting point. Carefully remove the clip and pinch the
end between your fingers. Let go of the balloon.
Answer the following questions:
a. What happens if you move the anchor points farther apart?
b. Record the results and determine the distance your balloon can travel.
c. Change how much you inflate the balloon. How does that affect the results?

Author: Shekaina Faith C. Lozada


School/Station: Gamut National High School
Division: Surigao del Sur
email address: shekainah.lozada@deped.gov.ph
d. What happen when you remove the clip?
e. What does the activity demonstrate?

Activity 3: Classify Me!

Objective: Classify daily activities into static and kinetic friction.


What you need:
 Paper and pen
What to do: Using the table below, classify the situations whether the friction force present is
static or kinetic. Write your answers on a separate sheet of paper.
1. Rolling a ball on a smooth pathway.
2. Pushing that box of books on the table but didn’t move.
3. Food in the table.
4. Your sister standing in front of your door.
5. A car slams on its brakes and skids to a stop.
6. A face towel hanging on a wall.
7. A bookmark in a book.
8. A car park on the hillside.
9. Pushing a loaded box across the floor.
10. Your father pulling a heavy table.

KINETIC FRICTION STATIC FRICTION


A car slams on its brakes and skids to a Rolling a ball on a smooth pathway
stop
Pushing a loaded box across the floor Pushing that box of books on the table
but didn’t move
Your father pulling a heavy table Food in the table
Your sister standing in front of your door
A face towel hanging on a wall
A bookmark in a book
A car park on the hillside

Reflection
You are sitting in a car, at rest, when another car rear ends your vehicle. Why do you
and the passengers experience a whiplash, in terms of Newton's first law of motion?

Rubrics
5 4 3 2 0
Practical application Practical application Practical Practical No
is scientifically is scientifically application is applications are discussion at
explained consistent explained consistent explained explained all.
to the concepts, and to the concepts, but consistent to the consistent to the
has no with minimal concepts but with concepts but with
misconceptions. misconceptions. one or two more than two
misconceptions. misconceptions.

References for learners:

Caintic, Helen. General Physics 1 for Senior High School 1st ed. C&E Publishing, Inc. 2017
Glancolli, Douglas. Physics Principles and Applications 6th ed. New Jersey: Pearson
Education, Inc. 2005.
Halliday, David and Resnick, Robert. Fundamentals of Physics 10th ed. John Wiley &
Sons, Inc. 2014.
Serway, Raymond and John Jewett. Physics for Scientists and Engineers 6th ed.

Author: Shekaina Faith C. Lozada


School/Station: Gamut National High School
Division: Surigao del Sur
email address: shekainah.lozada@deped.gov.ph
Thomson Brooks/Cole, 2004.
Young, Hugh D., Roger A. Freedman, A. Lewis Ford, and Hugh D. Young. Sears and
Zemansky's University Physics. 13th ed. Boston, MA: Pearson Learning Solutions,
2012.
Answer Key

Activity 1 B:

Activity 1 A:

Activity 3:

Activity 2:

Author: Shekaina Faith C. Lozada


School/Station: Gamut National High School
Division: Surigao del Sur
email address: shekainah.lozada@deped.gov.ph
WEEKLY LEARNING ACTIVITY SHEETS
General Physics 1, Grade 12, Quarter 1, Week 5

APPLICATION OF NEWTON’S LAW AND MECHANICAL WORK

Name: ___________________________________________ Section: ____________________

Most Essential Learning Competencies:


- Apply Newton’s 2nd law and kinematics to obtain quantitative and qualitative
conclusions about the velocity and acceleration of one or more bodies, and the contact
and noncontact forces acting on one or more bodies. (STEM_GP12N-Ie-36)
- Solve problems using Newton’s Laws of motion in contexts such as, but not limited to,
ropes and pulleys, the design of mobile sculptures, transport of loads on conveyor belts,
force needed to move stalled vehicles, determination of safe driving speeds on banked
curved roads. (STEM_GP12N-Ie-38)
- Calculate the dot or scalar product of vectors. (STEM_GP12WE-If-40)
- Determine the work done by a force acting on a system (STEM_GP12WE-If-41)

Specific Learning Objectives:

1. Distinguish the forces such as frictional force, normal force, threshold angles for
sliding, acceleration, etc., and mechanical work done by a force acting on a system.
2. Identify and draw the different forces acting on the system.
3. Apply Newton’s laws to solve problems involving the forces that act on an
accelerating body and determine work done by a force acting on a system
quantitatively.

Time Allotment: 4 hours

Key Concept
 Weight is a force on an object due to gravity. It is a vector quantity, w = mg; where m is
the mass of the object and g is the acceleration due to gravity. (Reminder: the
acceleration provided by gravity due on Earth is 9.80m/s2).
 The weight is dependent on the object’s location.
 The unit of weight is the same as the unit of force, N (newton).
If,
Fnet = ma
Weight is
w = mg
o Example: If John’s mass is 58.0kg, the magnitude of his weight is
w = mg
w = (58.0kg)(9.80m/s2)
w = 568.4kg. m/s2 or 568.4N
 Tension is a force on a rope, thread, string or cord. It is a pulling force and is present
only when the cord is taut or stretched. The direction of tension is always along the
string or rope and away from the object it supports.

Reference: https://www.khanacademy.org/science/physics/forces-newtons-laws/tension-tutorial/a/what-is-tension

Author: John Ferry Patosa Sual,MST


School/Station: Tigao National High School
Division: Surigao del Sur
email address: johnferry.sual@deped.gov.ph
Free-body Diagram
In physics and engineering, a free body diagram (force diagram, FBD) is a
graphical illustration used to visualize the applied forces, moments, and
resulting reactions on a body in a given condition.

T, tension

25kg w, weight

One-dimensional equilibrium: Tension in a rope with mass


Using Newton’s law for this stationary object we have
ΣF = T – w = 0, then
T = w = mg
T = (25kg)(9.80m/s2)
T = 245 N

But what if the same object is raised of a certain average acceleration.


What is the tension on the rope?

T, tension

a a, acceleration

25kg w, weight

Then, Newton’s equation maybe written as (assume that object accelerates by


2m/s2 the moment the rope was pulled through a pulley)
ΣF = T – w = ma
T = ma + w
T = ma + mg
T = (25kg)(2.00m/s2) + (25kg)(9.8 m/s2)
T = 50 N + 245 N
T = 295 N

Two-dimensional equilibrium: Atwood machine


The Atwood machine is
commonly used in the
constructions in lifting heavy
loads, theatre stages, etc.

In this case, we ignore the


friction on the pulley and focus
on the two objects of different
masses.

Author: John Ferry Patosa Sual,MST


School/Station: Tigao National High School
Division: Surigao del Sur
email address: johnferry.sual@deped.gov.ph
For this idealized case the tension “T” is the same on both sides of the pulley.
The acceleration “a” is the same for both masses. Solving for T gives:
T = m1g + m1a
Substituting T into the equation for m2 gives
m2g – m1g – m1a = m2a
The equation of motion for the two-mass system is then:
(𝑚2 − 𝑚1 )𝑔
(m2-m1)g = (m1+m2)a or 𝑎 =
(𝑚1 + 𝑚2 )

Example:

Curious about the topic of apparent weight that they discussed in class
recently, a 450-N physics student stands on a bathroom scale in an 850-kg
(including the student) elevator that is supported by a cable. As the elevator
starts moving the scale reads 400N. (a) What is the acceleration of the elevator?
(b) What is the tension in the cable?

Given:
w(student) = 450 N
m(student + bathroom) = 850 N
n = 400 N
a=?
T=?

Solution:

Author: John Ferry Patosa Sual,MST


School/Station: Tigao National High School
Division: Surigao del Sur
email address: johnferry.sual@deped.gov.ph
(a) Acceleration? (b) Tension?
Since the equation to solve for “a” is The tension in the cable can be
Fnet = n – w = ma and obviously we calculated using Newton’s Second
do not have the value for the Law
m(student). So we will calculate first the Fnet = T – w = ma
mass of the student given that T – mg = ma
student’s weight is 450 N, his mass T = ma + mg
can be calculated as T = m(a+g)
𝑚 𝑚
w = mg =(850kg)(−1.1 2 + 9.80 2)
𝑤 450 𝑁 𝑠 𝑠
m= = 𝑚 = 45.92 ≅ 46.0 kg =(850kg)(8.7𝑠 2)
𝑚
𝑔 9.80 2
𝑠 𝑚
= 7,395kg. 𝑠 2
Analysing the motion of the student we = 7,395 N
have
Fnet = n – w = ma
𝑛−𝑤 400 𝑁−450 𝑁 𝑚
a= = = -1.1 2
𝑚 46𝑘𝑔 𝑠

 Normal Force (N) is a kind of force exerted by the surfaces. The "normal" here refers to
perpendicular. This is because the normal force, usually represented with Fn or just N
is a force that is directed perpendicular to the two surfaces in contact.
Observe the figure below. What can you infer?

The card table sags when the dog food is placed


on it, much like a stiff trampoline. Elastic
restoring forces in the table grow as it sags
until they supply a normal force Fn or just N
equal in magnitude and opposite in direction to
FBD the weight of the load. (Image Credit: Openstax
College Physics)

 Since no horizontal motion is observed then ΣFx = 0.


 The vector sum of the vertical forces does not make the object rise nor
sink so, ΣFy = N – w = 0 and N = w = mg

o However, there are instances that the normal force is not equal to the weight of
the mass, consider the example below:

The figure shows an object in an


inclined plane, what would be the
Note: w = mg
FBD in this type of situation?
 Reference: https://www.sarthaks.com/745825/explain-the-motion-of-a-body-on-an-inclined-plane
 Take note in this situation we ignore the force due to friction.

Author: John Ferry Patosa Sual,MST


School/Station: Tigao National High School
Division: Surigao del Sur
email address: johnferry.sual@deped.gov.ph
 To determine the components of the forces along and perpendicular to
the plane, we have:
o wx = wsinϴ (wx is the component of weight along parallel to the surface)
o wy = wcosϴ (wy is the component perpendicular to the surface)
 Thus, the vector sum of the forces along y is zero since the object
neither rises nor sinks.
ΣFy = 0
N - wcosϴ = 0
N = wcosϴ
 However, along the x-axis, there is only one force: the x-component of
the weight. This is the force that will make the object slide itself.
ΣFx = max
ΣFx = wsinϴ
wsinϴ = mg sinϴ
ax = g sinϴ

Example:

In a field trip to an ice plant, you noticed a 5.00kg block of ice was
released from rest at the of a 2.00m long frictionless ramp with an angle of 13.3 0.
It slides downhill, reaching a speed of 3.00m/s at the bottom. (a) Draw the free-
body diagram of the block of ice. (b) What is the acceleration of a block of ice?

Given:
m(ice)= 5.0kg
vf = 3.00m/s
ϴ = 13.30
a=?

Solution:
(a) FBD (b) Acceleration?
We calculate acceleration
through this equation
ΣFalong x = wx – F = max
wx – F = max
m m
𝑤𝑥 −𝐹
ax =
𝑚
𝑚𝑔𝑠𝑖𝑛ϴ−𝐹
ax =
𝑚
9.8𝑚 0
(5.0𝑘𝑔)( 2 )𝑠𝑖𝑛13.3 −10.0𝑁
ax = 𝑠
5.0𝑘𝑔
ax = 0.26 m/s2

Author: John Ferry Patosa Sual,MST


School/Station: Tigao National High School
Division: Surigao del Sur
email address: johnferry.sual@deped.gov.ph
 Friction is a resisting force. Its direction is opposite to that of the motion.

(a)
Push on the car by the
man

Friction opposing the


motion
Reference: https://physics.stackexchange.com/questions/488084/why-a-box-moving-
at-constant-velocity-isnt-doing-

 Static Friction (fs) is the force that keeps an object “stuck” on a surface and
prevents its motion. Its magnitude is given by the equation:
fs = µsn
whereas: µs is the coefficient of static friction
n is the normal force
 Kinetic Friction (fk) is the force that “opposes the motion.” This is for the
objects that are moving relative to the point of contact. Its direction is
opposite to the direction of motion. The magnitude of the friction force is
given by:
fk = µkn
whereas: µk is the coefficient of kinetic friction
n is the normal force

Example:
A 1,500 kg minicar is pulled with a force of 11.0kN at an angle of 30.0 0
from the horizontal. If the coefficient of friction between the tires and the road is
0.800, what is the average acceleration of the car?

Given: Solution:
m(car) = 1,500 kg The frictional force is computed thru this equation:
F = 11.0kN Ff = µ(mg - Fsinϴ)
ϴ = 30.00 = 0.8[ (1,500kg)(9.80m/s2) – (11,000 N sin30.00 )]
µ = 0.800 = 7,360 N
a=?
Acceleration?
ΣFnet = Fcos ϴ - Ff = ma

Author: John Ferry Patosa Sual,MST


School/Station: Tigao National High School
Division: Surigao del Sur
email address: johnferry.sual@deped.gov.ph
Fcos ϴ − F𝑓 11,000Ncos30.00 − 7,360N
a= 𝑚
= = 1.44m/s2
1,500𝑘𝑔

 Work
 Work is defined as “the product of force and displacment’.
 Work is a scalar product, so it has a maximum value when force and
displacement are PARALLEL to each other (ϴ = 00)
 Thus, in equation form, it is written as :
W = F.d or W = Fdcos ϴ
 The S.I. unit for W is J (joule) or N.m or kg.m2/s2

Example (1):
A force of 3.00N acts through a distance of 12m in the direction of the
force. Find the work done?
Given: Solution:
F = 3.00 N W = Fdcos ϴ
d = 12m = (3.00 N)(12m)cos00
ϴ = 0 (0 since the F and d are parallel)
0
= (3.00 N)(12m)(1)
W=? = 36 N.m or 36 J

Example (2):
A 2kg book is held 1 m above from the ground for 50 s. What is the work
done on the book?
Given: Solution:
m = 2kg W = Fdcos ϴ
d = 1m = mg. dcos ϴ
ϴ = 90 (90 since the F and d are
0
= [(2kg)(9.8m/s2)(1m)cos900
perpendicular with each other) = (19.6 N)(1m)(0)
F = ma, however the force = 0 N.m or 0 J
acting in this is weight (w = mg)
W=?

Activity 1: Let’s Play: Who’s Telling the Truth!

Objective: Distinguish the forces such as frictional force, normal force, threshold angles for
sliding, acceleration, etc., and mechanical work done by a force acting on a system.

What you need:


 Paper and pen

What to do: Write True if the statement is correct and write False if the statement is wrong.
______1. The amount of matter in an object is called weight.
______2. The SI unit of force is called the kilogram.
______3. If a hockey puck slides on a perfectly frictionless surface, it will eventually slow down
because of its inertia.
______4. A force applied on a sturdy wall produces no work at all.
______5. The combination of all the forces that act on an object is called the net force.
______6. Tension is the result of opposite forces in a connector, such as a string, rope, chain or
cable, that pulls each point of the connector apart in the direction parallel to the length of the
connector. At the ends of the connector, the tension pulls toward the center of the connector.
______7. Normal reaction is the force that opposes the force of gravity and acts in the direction
of the force of gravity.
______8. When two unequal forces act on a body, the body will not move in the direction of the
weaker force.

Author: John Ferry Patosa Sual,MST


School/Station: Tigao National High School
Division: Surigao del Sur
email address: johnferry.sual@deped.gov.ph
______9. Carrying a load of books a distance of 5m produces no work at all.
______10. If all other variables are equal, then an object with a greater mass would have a
more difficult time accelerating.

Activity 2: Give me Direction!

Objective: Identify and draw the appropriate free-body diagram in every picture.

What you need:


 Paper, pencil, pen or colour pen.

What to do: Draw the appropriate free-body diagram in every picture and label each vector
with whether T (tension), Ff (friction force), N (normal force), and w (weight).

1. Pulling a car 2. A rolling metal ball

3. Hanging Picture frame 4. A man sweeping the floor

5. A truck towing a car (only the car being pulled/towed)

Author: John Ferry Patosa Sual,MST


School/Station: Tigao National High School
Division: Surigao del Sur
email address: johnferry.sual@deped.gov.ph
Activity 3: Solve Me!

Objective: Solve problems involving the different forces acting on a system governed by the
Laws of Newton.

What you need:


 Paper and pen
 Scientific Calculator

What to do: Answer the following problems. Write your solutions in a separate sheet of paper.

1. A 20-kg crate is lowered down a plane inclined at an angle of 370 with the horizontal by
a rope that exerts on the crate a 50-N force directed parallel to the plane. A 40-N
friction force, also parallel to the plane, opposes the downward motion. What is the
acceleration crate?

2. Curious about the topic of apparent weight that they discussed in class recently, a 450-
N physics student stands on a bathroom scale in an 850-kg (including the student)
elevator that is supported by a cable. As the elevator starts moving the scale reads
400N. (a) What is the acceleration of the scale reads 550N? (b) What is the tension in
the cable for this case?

3. It takes 250 N of horizontal to set a 40 kg box in motion across a floor. Once the box is
in motion, a 305 N horizontal force keeps it moving at a constant velocity. Calculate the
coefficient of dynamic friction between the box and the floor.

4. A child pushes a toy box 5.0 m along the floor with a force of 7.0 N directed downward
at an angle of 30.00 to the horizontal. (a) How much work does the child do? (b) Would
you expect more or less work to be done if the child pulled upward at the same angle?

Reflection

How will you have competitive advantage in sports upon applying Newton’s Laws of
motion? Write your 5-sentence answer in a separate sheet of paper.

Rubric for Reflection


3 2 1
Practical application is Practical application is Practical application is
scientifically explained consistent scientifically explained consistent explained consistent to the
to the concepts, and has no to the concepts, but with minimal concepts but with
misconceptions. misconceptions. misconceptions.

References:
Books
Albelda, J. (c.2013). Mechanics pp. 37-40. Philippine Normal University-The National
Center for Teacher Education.Manila, Philippines.

Coronado, G., & Boncodin, M., (c.2006). Newton’s law of Motion pp.60-71. Phoenix
Publishing House, Inc.

Coronado, G., & Boncodin, M., (c.2006). Work, Power, and Energy pp.95-96. Phoenix
Publishing House, Inc.

Author: John Ferry Patosa Sual,MST


School/Station: Tigao National High School
Division: Surigao del Sur
email address: johnferry.sual@deped.gov.ph
Department of Education. Newton’s Laws of Motion and Application p. 106. General
Physics 1-Learner’s Material First Edition 2018.

Department of Education. Work, Energy, and Power pp.73-104. General


Physics 1-Learner’s Material First Edition 2018.

Internet
https://en.wikipedia.org/wiki/Free_body_diagram

http://hyperphysics.phy-astr.gsu.edu/hbase/atwd.html

https://www.khanacademy.org/science/physics/forces-newtons-laws/normal-contact-force/a/what-is-
normal-force

https://www.sfu.ca/~mxchen/phys1010901/LectureCh05.pdf

http://www.physics.usyd.edu.au/~helenj/Mechanics/PDF/mechanics04.pdf

https://physics.stackexchange.com/questions/488084/why-a-box-moving-at-constant-velocity-isnt-doing-
work

http://www.wscacademy.org/ourpages/auto/2012/12/2/42640104/Physics%20Chapter%203%20Practice
%20Test.pdf

https://openstax.org/books/physics/pages/5-introduction

Answer Key

Activity 2: Give me Direction!


10. True
9. True
8. False
7. False
6. False
5. True
4. True
3. True
2. False
1. False
Who’s Telling the Truth!
Activity 1: Let’s Play:

1. 2.

3.
4.

Author: John Ferry Patosa Sual,MST


School/Station: Tigao National High School
Division: Surigao del Sur
email address: johnferry.sual@deped.gov.ph
email address: johnferry.sual@deped.gov.ph
Division: Surigao del Sur
School/Station: Tigao National High School
Author: John Ferry Patosa Sual,MST
5.
Activity 3: Solve me
1. 1.39m/s2
2. a. 2.2 m/s2
b. 10, 000N
3. 0.7781 ≅ 0.78
4. A. 30.31 J
5. B. More work will be
done since another
presence of force
will be considered, w
(weight)

You might also like